Bar Review Pointers Evidence 2015

January 10, 2018 | Author: Bi Tin | Category: Hearsay In United States Law, Hearsay, Evidence (Law), Judiciaries, Virtue
Share Embed Donate


Short Description

Bar Review Pointers Evidence 2015...

Description

JURISTS BAR REVIEW CENTER™ BAR REVIEW POINTERS ON EVIDENCE* Prof. Manuel Riguera

31 May 2015

EVIDENCE. The means, sanctioned by the Rules of Court, of ascertaining in a judicial proceeding the truth respecting a matter of fact. (S1 R128). Evidence also refers to the medium used to prove the existence or non-existence of a fact in issue. While administrative or quasi-judicial bodies, such as the Office of the Ombudsman, are not bound by the technical rules of procedure, this rule cannot be taken as a license to disregard fundamental evidentiary rules; the decision of the administrative agencies and the evidence it relies upon must, at the very least be substantial. In an investigation before the OMB for grave misconduct, the SC upheld the inadmissibility of affidavits and NBI report based thereon on the ground of hearsay. (Miro v. Vda de Erederos, 20 November 2013). SCOPE. The rules of evidence shall be the same in all courts and in all trials and hearings, except as otherwise provided by law or by the Rules of Court (S2 R128) or by Supreme Court issuances. For instance under the Rules on Summary Procedure, in criminal cases the direct testimonies of witnesses shall be in affidavit form subject to cross-examination. ADMISSIBILITY. Evidence is admissible when it is relevant to the issue and is not excluded by the law or by the Rules of Court. (S3 R128). ADMISSIBILITY = RELEVANCY + COMPETENCY. RELEVANCY Evidence is relevant when it has a relation to the fact in issue as to induce belief in its existence or nonexistence. (iben) Collateral matters: matters which have no direct connection to the fact in issue. Thus in a suit on a promissory note by P against D, the fact that D did not pay A is a collateral matter. Is evidence on collateral matters allowed? No except when it tends in any reasonable degree to establish the probability or improbability of the fact in issue. (S4 R128) (trepif). COMPETENCY Even if the evidence is relevant, but it is excluded by the law or by the Rules of Court, the evidence would be inadmissible. Exclusionary rules are present in the Rules of Court, for example those relating to the various disqualification rules and privileges. They may also be found in the Constitution such as under Sections 2 and 3, Article III (right v. unreasonable search and seizure, right to privacy of communication and correspondence), Section 12, Article III (rights of person under custodial investigation), Section 17, Article III (right v. self-incrimination), in statutes, such as the bank secrecy law (R.A. No. 1405) and the Anti-Wiretapping Law (R.A. No. 4200). Q In a disbarment case, introduced in evidence against the respondent was a birth certificate showing that he had fathered children by his mistress. The respondent invokes Article 7 of P.D. No. 603 which provides that the birth records of a person shall be kept strictly confidential and that no information thereto shall be disclosed except on request of the person himself or of a court or proper government *

Cases surveyed up to the period ending 31 March 2015.

Jurists Bar Review Pointers on Evidence 2015. All rights reserved 2015 by Jurists Review Center, Inc. Unauthorized reproduction, use, or dissemination strictly prohibited and shall be prosecuted to the full extent of the law, including administrative complaints with the Office of the Bar Confidant Supreme Court. Bar Review Pointers on Evidence by Prof. Manuel R. Riguera for Jurists Bar Review Center

1

official and which punishes with imprisonment and/or fine any unauthorized disclosure. Is the birth certificate admissible in evidence? A Yes. Article 7 of P.D. No. 603 only provides for sanctions against persons violating the rule on confidentiality of birth records, but nowhere does it state that procurement of birth records in violation of said article would render said records inadmissible in evidence. On the other hand, the Rules of Evidence only provides for the exclusion of evidence if it is obtained as a result of illegal searches and seizures. It should be emphasized, however, that said rule against unreasonable searches and seizures is meant only to protect a person from interference by the government or the state. Consequently, in this case where complainants, as private individuals, obtained the subject birth records as evidence against respondent, the protection against unreasonable searches and seizures does not apply. Since both Article 7 of P.D. No. 603 and the Revised Rules on Evidence do not provide for the exclusion from evidence of the birth certificates in question, said public documents are, therefore, admissible and should be properly taken into consideration in the resolution of this administrative case against respondent. (Tolentino v. Mendoza, 440 SCRA 519 [2004], cited in De Jesus v. Sanchez-Malit, 8 July 2014). KINDS OF EVIDENCE (DOT) DOCUMENTARY EVIDENCE. Writings or any material offered as proof of their contents. OBJECT (OR REAL) EVIDENCE. Evidence which is directly addressed to the court’s senses. TESTIMONIAL EVIDENCE. Consists of the statement of a witness offered to the court. Maybe oral/live or written (affidavit). DOCUMENTARY EVIDENCE. Writings or any material containing letters, words, numbers, symbols, or other modes of written expression offered as proof of their contents. Hence a document can be object evidence if offered not as proof of its contents but of its existence or physical condition or features. BEST EVIDENCE RULE (ORIGINAL DOCUMENTS RULE) S3-7 R130 Presently we have two best evidence rules. One under S3 R130 and another one under R4 REE. Best evidence rule under S3 R130 When the subject of inquiry is the contents of a document, no evidence shall be admissible other than the original document itself. (S3 R130). BER applies only to documentary evidence not to testimonial/oral or object evidence. Air France v. Carrascoso, 18 SCRA 155. Testimony on what the purser read to the witness from a notebook not covered by BER. People v. Tandoy, 192 SCRA 28. Photocopies of buy-bust money admissible in evidence. WHAT IS THE ORIGINAL? 1. 2. 3.

Original simply means the document the contents of which are the subject of inquiry. (csi) (S4(a) R130). Duplicate originals. Regularly repeated business entries.

Exceptions to the BER: (CLAP) C - Custody/Control. Original is in the custody or control of the adverse party and he fails to produce despite reasonable notice.

Jurists Bar Review Pointers on Evidence 2015. All rights reserved 2015 by Jurists Review Center, Inc. Unauthorized reproduction, use, or dissemination strictly prohibited and shall be prosecuted to the full extent of the law, including administrative complaints with the Office of the Bar Confidant Supreme Court. Bar Review Pointers on Evidence by Prof. Manuel R. Riguera for Jurists Bar Review Center

2

L - Lost/Destroyed/Production cannot be made. (LDP). Note that the law requires from the offeror absence of bad faith not of negligence. A - Accounts which are numerous and cannot be examined without great loss of time and the fact sought to be established is only the general result of the whole. (NTG grew). A witness need not present the original accounts but can present a summary thereof. P - Public record or recorded in a public office. May be proved by a certified true copy issued by the public officer in custody thereof. Trial court’s discretion to dispense with production of original Estrada v. Desierto, 356 SCRA 108 (2001). The SC adopted Wigmore’s comment that production of the original may be dispensed with, in the trial court’s discretion, if the opponent does not bona fide dispute the contents of the document and no other useful purpose will be served by requiring production. (DBU). Gaw v. Chua, G.R. No. 160855, 16 April 2008, Nachura, J. Production of the original may be dispensed with in the trial court’s discretion whenever the opponent does not bona fide dispute the contents of the documents and no other useful purpose will be served by requiring production. Here there was no dispute as to the terms of either deed; hence the RTC correctly admitted the photocopies in evidence. Concepcion Chua admitted signing the deed of partition and in effect admitted the genuineness and due execution of the deed of sale when she failed to specifically deny it under oath pursuant to S8 R8. The Spouses Gaw did not contest the contents of the deeds but simply alleged that there was a contemporaneous agreement that the transfer of Hagonoy Lumber to Chua Sioc Huan was only temporary. A certification by an officer of the local assessor’s office that a copy of the deed of sale is a certified true copy is not an acceptable secondary evidence. First the local assessor’s office is not the official repository of original notarized deeds of sale and could not have been the legal custodian contemplated in S24 R132. Second the certification did not state that it is a certified true copy of the original. (Gabatan v. CA, 13 March 2009). The fact that the articles of incorporation, memorandum of agreement, purchase agreement, confidential memorandum, and letters were collected by the PCGG in the course of its investigation of the alleged illgotten wealth of the Marcoses did not make them public records within the purview of S3(c) and S7 R130. Hence they could not be admitted for being in violation of the best evidence rule. As to the copy of the TSN of the proceedings before the PCGG, while it may be considered as a public document since it was taken in the course of the PCGG’s exercise of its mandate, it was not attested to by the legal custodian to be a correct copy of the original. This omission falls short of the requirement of Rule 132, Secs. 24 and 25 of the Rules of Court. (Republic v Marcos-Manotoc, 8 February 2012). Presentation of photocopy of Gulf Air finance manual permissible in labor proceedings which are nonlitigious. (Gulf Air v. NLRC, 24 April 2009). Best evidence rule does not apply where the contents of a document are not the subject of inquiry, as in the case where neither party disputed the contents of a service contract. (Nissan North EDSA v. United Phil. Scout Agency, 20 April 2010). Proper foundation for the introduction of secondary evidence 1. Proof of the execution or existence of the document. 2. Proof of the cause of its unavailability. Secondary evidence (CRT) 1. Copy. 2. Recital in an authentic document. 3. Testimony of a witness.

Jurists Bar Review Pointers on Evidence 2015. All rights reserved 2015 by Jurists Review Center, Inc. Unauthorized reproduction, use, or dissemination strictly prohibited and shall be prosecuted to the full extent of the law, including administrative complaints with the Office of the Bar Confidant Supreme Court. Bar Review Pointers on Evidence by Prof. Manuel R. Riguera for Jurists Bar Review Center

3

Proponent must account for non-production of all the originals Citibank v. Teodoro, G.R. 150905, 23 Sep 03. Suit by the Petitioner against the Respondent for credit card debts, at issue was the admissibility of photocopies of sales invoices/charge slips signed by the Respondent. SC said the proponent must account for the non-production of all the originals. Where the loss of the marriage certificate has been proven, secondary evidence in the form of testimonies may be adduced to prove the fact of marriage. (Vda. de Avenido v. Avenido, 22 January 2014). BEST EVIDENCE RULE UNDER THE REE The Rules on Electronic Evidence (REE) took effect on 1 August 2001. Please take note that its coverage was extended to criminal actions and proceedings effective 14 October 2002. (A.M. No. 01-701-SC, dated 24 September 2002). The REE shall apply whenever an electronic document is used or offered in evidence. Electronic Document: Information or data or representation of information or data which is received, recorded, transmitted, stored, processed, retrieved, or produced (3RP TSP) by electronic, optical, or similar means by which a right is established or an obligation extinguished, or by which fact may be proved. (R2(g)&(h), REE). It includes digitally signed documents and print-outs. (Id.). The REE applies to electronic, not non-electronic, documents. The term original includes the soft copy and the printouts or outputs (hard copy) readable by sight or other means (S1 R4 REE). Copies or duplicates, that is those reproduced by mechanical, electronic, or chemical means or by other equivalent techniques which accurately reproduces the original, shall be regarded as the equivalent of the original, Unless (AU)  a genuine question is raised as to the authenticity of the original; or  in the circumstances it would be unjust or inequitable to admit the copy in lieu of the original. Q Is the Best Evidence Rule under the Rules on Electronic Evidence applicable to non-electronic documents? A No. (MCC Industrial Corporation v. Ssangyong Corp., G.R. No. 170633, 17 October 2007). Hence non-electronic documents (paper-based documents) are covered by the best evidence rule under S3 R130. Best Evidence Rule (S3 R130) Best Evidence Rule (R4 REE) Applies to non-electronic & electronic Applies to electronic documents only documents Copies or duplicates not equivalent of Copies or duplicates equivalent of the original original Photocopies of fax transmissions are not admissible unless the non-production of the original fax transmission is accounted for. Fax transmissions are not electronic documents or electronic evidence. (MCC Industrial Corporation v. Ssangyong Corp., 17 October 2007, Nachura, J.) If not all the contents of a document are generated or recorded electronically, such as if the document contains a manual signature and handwritten notations, then a photocopy of such document will not be considered as an equivalent of the original. (National Power Corporation v. Codilla, 3 April 2007). Text messages are ephemeral electronic communications. Once saved or recorded however they become electronic documents and are subject to the best evidence rule.

Jurists Bar Review Pointers on Evidence 2015. All rights reserved 2015 by Jurists Review Center, Inc. Unauthorized reproduction, use, or dissemination strictly prohibited and shall be prosecuted to the full extent of the law, including administrative complaints with the Office of the Bar Confidant Supreme Court. Bar Review Pointers on Evidence by Prof. Manuel R. Riguera for Jurists Bar Review Center

4

PAROL EVIDENCE RULE (S9 R130) Once an agreement has been reduced to writing, parol or extraneous evidence (oral or written) may not be admitted to modify, explain, or add to the terms of the written agreement. The PER applies only to written contracts. Thus a receipt which is merely proof of payment is not covered by the PER. A will is considered a written agreement for purposes of the PER. The SC held that the parol evidence rule applies only to the parties to the contract and their successorsin-interest and not to the defendants who are not parties to the deed of sale. (Lechugas v. CA, 143 SCRA 355). EXCEPTIONS: VISA (These must be put in issue) V – Validity of written agreement put in issue. I – Intent of parties not expressed in written agreement. S – Subsequent agreements, i.e, made after the execution of written agreement. A – Ambiguity (latent), imperfection, mistake. (AIM). Inciong v. CA, 257 SCRA 578 (1996). PER applies to written contracts be they private or public documents. The contract need not be signed by two parties nor need it be bilateral. Ortañez v. CA, 266 SCRA 561. The SC stated that the existence of an exception to the PER must be “squarely presented as an issue” or otherwise expressly averred. E.g., “failed to express the true agreement of the parties” not merely that “the sale was subject to four conditions.” Implicit averment not sufficient. The SC also clarified when parol evidence of conditions precedent may be admitted, i.e., when the document mentions such condition or refers to an agreement containing such condition. Even if the deed of sale stated that the buyers had paid the consideration, the seller may prove otherwise. This is an exception to the parol evidence rule, that is, the failure of the written agreement to express the true intent of the parties. (Sps. Lequin v. Sps. Vizconde, 12 October 2009). A beneficiary under a stipulation pour autrui is considered a party to a written contract and is bound by the parol evidence rule. (Pacres v. Heirs of Ygona, 5 May 2010). Rudlin Corp contends that although the price stated in the contract is P6.9 million, there was an understanding that the price would be reduced to P6 million. They invoke the exception to the PER under S9(a) R130 wherein the written agreement failed to express the true intent of the parties. SC held that S9(a) R130 available only where written contract is so ambiguous or obscure that the parties’ contractual intention cannot be understood from a mere reading of the instrument. (Financial Building Corp. v. Rudlin International Corp., 4 October 2010, Villarama, J.) SPECIAL KINDS OF EVIDENCE EPHEMERAL ELECTRONIC COMMUNICATION. Ephemeral Electronic Communication. Refers to phone talks, text msgs, chatroom sessions, streaming audio and video (PTC SS), and other forms of electronic communication the evidence of which is not recorded or retained. (S2(k) R2, REE). If recorded by audio, photo or video means, becomes audio, photo or video evidence. If recorded as an electronic document, becomes such. A text message which is saved becomes an electronic document. The Supreme Court recently admitted text messages as evidence against a CA personnel who was accepting bribes from a litigant. Electronic documents as functional equivalent of paper-based document. Whenever a rule of evidence refers to a writing, document, record, etc., such a term is deemed to include an electronic document.

Jurists Bar Review Pointers on Evidence 2015. All rights reserved 2015 by Jurists Review Center, Inc. Unauthorized reproduction, use, or dissemination strictly prohibited and shall be prosecuted to the full extent of the law, including administrative complaints with the Office of the Bar Confidant Supreme Court. Bar Review Pointers on Evidence by Prof. Manuel R. Riguera for Jurists Bar Review Center

5

As to the admissibility of the text messages, the RTC admitted them in conformity with the Court’s earlier Resolution applying the Rules on Electronic Evidence to criminal actions.1 Text messages are to be proved by the testimony of a person who was a party to the same or has personal knowledge of them. Here, PO3 Cambi, posing as the accused Enojas, exchanged text messages with the other accused in order to identify and entrap them. As the recipient of those messages sent from and to the mobile phone in his possession, PO3 Cambi had personal knowledge of such messages and was competent to testify on them. (Pp v Enojas, 10 March 2014). DEOXYRIBONUCLEIC ACID/DNA. The chain of molecules found in every nucleated cell of the body. The totality of an individual’s DNA is unique for the individual, except identical twins. DNA testing is conclusive proof of non-paternity. If the value of the Probability of Paternity is less than 99.9%, the results of the DNA testing shall be considered as corroborative evidence; if 99.9% or higher, there shall be a disputable presumption of paternity. (Rule on DNA Evidence). First application of New Rules on DNA Evidence. Biological samples taken from AAA (rape victim), BBB (17 y.o.), and the accused. Results showed that there is a 99.999% probability that the accused is the father of BBB. (People v. Umanito, 16 April 2009). While the accused’s DNA profile was not in the vaginal smear, the DNA result was not conclusive since the specimen was contaminated or stained, having undergone a serological analysis. (People v. Pascual, 19 January 2009). Compliance with the conditions under S4 of the Rule on DNA Evidence does not mean that a DNA testing order will be issued as a matter of right. During the hearing on the motion for DNA testing, the petitioner must present prima facie evidence or establish a reasonable possibility of paternity. (Lucas v. Lucas, 6 June 2011). QUALIFICATION OF WITNESSES Any person who can perceive and make known his perception to others is qualified to be a witness. A witness does not need to be qualified beforehand as there is a presumption that the witness is qualified to be such. There is no law requiring a witness to present authorization to testify from the party presenting him. All that the Rules require is that a witness possess all of the qualification and none of the disqualification provided therein. (AFP-RSBS v Republic, 20 March 2013). A deaf-mute is not incompetent as a witness. All persons who can perceive, and perceiving, can make known their perception to others, may be witnesses. Deaf-mutes are competent witnesses where they (1) can understand and appreciate the sanctity of an oath; (2) can comprehend facts they are going to testify on; and (3) can communicate their ideas through a qualified interpreter. (Pp v Aleman, 24 July 2013). Child witness (S21(b) R130; Sec. 6, Rule of Examination of a Child Witness [RECW]). Every child is presumed qualified to be a witness. However the court may conduct a competency examination if substantial doubts exist as to the child’s competency to testify. Child witness: One who at the time of giving testimony is below 18. Also those 18 and above but unable to fully take care of himself or protect himself from cruelty, abuse, neglect, exploitation or discrimination (CANED) because of a physical or mental disability or condition. (S4a) Cavili v. Fernando, 154 S 610. Party declared in default may still testify for non-defaulted party.

This statement puts to rest the doubts created by the SC’s obiter in Ang v. Court of Appeals, G.R. No. 182835, 20 April 2010, that the REE does not apply to criminal actions. 1

Jurists Bar Review Pointers on Evidence 2015. All rights reserved 2015 by Jurists Review Center, Inc. Unauthorized reproduction, use, or dissemination strictly prohibited and shall be prosecuted to the full extent of the law, including administrative complaints with the Office of the Bar Confidant Supreme Court. Bar Review Pointers on Evidence by Prof. Manuel R. Riguera for Jurists Bar Review Center

6

DISQUALIFICATIONS OF WITNESSES (MIM DIP) -

Mental incapacity or immaturity. Marriage (Marital Disqualification Rule) Death or Insanity. (Dead Man’s Statute and claim v. insane) Privileged Communication (MAP PP)

DEAD MAN’S RULE (S23 R130) A party having a claim or demand against the estate of a deceased person or against his executor/administrator cannot testify as to fact occurring before the decedent’s death. -

One way street. Works only against person making a claim v. the estate of the deceased. (Tongco v. Vianzon, 50 P 698). Where the estate filed a counterclaim against the survivor-claimant, the latter can testify on antemortem facts in his defense. Witness other than the claimant may testify on antemortem facts. Applies only to testimonial evidence, not object or documentary evidence. Hence the claimant may present, identify, and offer in evidence a promissory note signed by the decedent.

Ong Chua v. Carr, 53 P 975. Survivor may testify against the deceased’s estate where the deceased was guilty of fraud. Fraud should however be established by independent evidence other than survivor’s testimony. “What the PARAD, DARAB and CA failed to consider and realize is that Amanda’s declaration in her Affidavit covering Pedro’s alleged admission and recognition of the alternate farming scheme is inadmissible for being a violation of the Dead Man’s Statute.” (Garcia v. Vda. De Caparas, 17 April 2013). What the SC in turn overlooked was that Amanda was not the party-claimant or assignor of a party to a case or one in whose behalf a case is prosecuted but only a witness; hence the Dead Man’s Statute does not bar her from testifying. MARRIAGE DISQUALIFICATION RULE (S22 R130) During the marriage, a spouse cannot testify for or against the other, except in a civil case by one against the other or a criminal case committed by one spouse against the other or the other’s ascendants or descendants. (SAD) - Marriage must be existing at the time of the offer of the testimony. Even if marriage occurred after the fact sought to be testified to. - Husband may bar wife from testifying for or against her. - Lezama v. Rodriguez, 23 S 1166. Where the husband and wife were charged with having colluded to defraud the plaintiff, the wife cannot be called as an adverse party witness by the plaintiff over the husband’s objection. Alvarez v. Ramirez, 14 October 2005: Where the husband had tried to set fire to his sister-in-law’s house knowing fully well that his wife was also inside, the wife could testify against him as he committed a crime against her. The SC also stated that where there is no more harmony to be preserved nor peace and tranquility to be disturbed, the reason for the MDR ceases and a spouse may testify against the other. PRIVILEGES (MAP PP) -

Marital Communication Privilege Attorney-Client Privilege Physician-Patient Privilege. Priest-Penitent Privilege Public Interest Privilege.

Jurists Bar Review Pointers on Evidence 2015. All rights reserved 2015 by Jurists Review Center, Inc. Unauthorized reproduction, use, or dissemination strictly prohibited and shall be prosecuted to the full extent of the law, including administrative complaints with the Office of the Bar Confidant Supreme Court. Bar Review Pointers on Evidence by Prof. Manuel R. Riguera for Jurists Bar Review Center

7

MARITAL COMMUNICATION PRIVILEGE (S24(a) R130) The husband or wife cannot be examined without the consent of the other as to any confidential communication received from the other during the marriage except in a civil case by one against the other or in a criminal case for a crime committed by one against the other or the latter’s ascendants or descendants. -

-

Unlike MDR, the MCR still holds even when the marriage has been terminated as S24(a) R130 states “during or after the marriage.” Privileged matter is limited though, that is, confidential communications received during the marriage. Thus a wife can testify as to a husband’s dying declaration as the same is not confidential. (U.S. v. Antipolo, 37 P 726. Where privileged communication comes into hands of third person without collusion, privilege does not apply.

The ones who can invoke the privilege are the spouses. Others cannot invoke it. ATTORNEY-CLIENT PRIVILEGE An attorney cannot, without the client’s consent, be examined as to any communication made by the client to him or his advice given thereon in the course of, or with a view to, professional employment. (CAP) -

-

Extends to attorney’s secretary, stenographer, or clerk. Paralegal should also be deemed covered. Cannot be examined without the consent of the client and the employer. The rules safeguarding privileged communication between attorney and client shall apply to similar communications made to or received by the law student, acting for a legal clinic. S3 R138-A (Law Student Practice Rule). Lawyer may reveal secrets when necessary to collect fees or to defend (cd) himself, his associates, or employees. (Rule 21.01(c), Code of Professional Responsibility).

Regala v. Sandiganbayan, 262 SCRA 124 (1996). Client’s identity privileged where it would implicate the client for the very activity which he sought the lawyer’s advice. People v. Sandiganbayan, 275 SCRA 505 (1997). Privileged information relates only to past crimes not intention to commit future crime. Cut-off point is when the communication was made to the client, not when witness will testify. Upjohn v. United States, 449 U.S. 383 (1981). In the case of a corporate client, the privilege extends not only to managerial but to rank-and-file employees as well. PHYSICIAN-PATIENT PRIVILEGE A person authorized to practice medicine, surgery, or obstetrics cannot in a civil case, without the patient’s consent, be examined as to any advice or treatment given by him or any information which he may have acquired in attending such patient in a professional capacity, which information was necessary to enable him to act in that capacity, and which would blacken the patient’s reputation. (PCATI PNB) -

Applies only to civil cases. Law and order considerations override the privilege in criminal cases. The information must blacken the patient’s reputation. So information concerning physical injuries which are not compromising may be inquired into. Autopsies conducted by medico-legal officers are not privileged as there is no patient involved.

Lim v. CA, 214 S 273. Attending psychiatrist could testify and give expert opinion where the same is based only on strictly hypothetical questions and not on information obtained while attending to patient. Jurists Bar Review Pointers on Evidence 2015. All rights reserved 2015 by Jurists Review Center, Inc. Unauthorized reproduction, use, or dissemination strictly prohibited and shall be prosecuted to the full extent of the law, including administrative complaints with the Office of the Bar Confidant Supreme Court. Bar Review Pointers on Evidence by Prof. Manuel R. Riguera for Jurists Bar Review Center

8

Krohn v. CA, 233 S 146, 14 June 1994: In a marriage annulment case, a husband could testify as to the contents of a confidential psychiatric evaluation of his wife as a schizophrenic. The SC said that the privilege bars only the physician not other persons. The SC said that the testimony may not be considered a circumvention of the privilege since the husband’s testimony cannot have the same force and effect as the testimony of the physician. The respondent’s counsel waived the hearsay objection when he failed to raise this as a ground in his objection. Chan v Chan, 24 July 2013: In an action for nullification of marriage, Josielene filed a motion for the issuance of a subpoena duces tecum to the hospital where Johnny had undergone rehabilitation for the production of his medical records. The SC treated the motion as one for production of documents under R27 (since trial had not yet started) which does not cover privileged matter. Josielene claimed that the hospital records subject of this case are not privileged since it is the "testimonial" evidence of the physician that may be regarded as privileged. The privilege, says Josielene, does not cover the hospital records, but only the examination of the physician at the trial. The SC held that to allow the disclosure during discovery procedure of the hospital records—the results of tests that the physician ordered, the diagnosis of the patient’s illness, and the advice or treatment he gave him—would be to allow access to evidence that is inadmissible without the patient’s consent. A physician memorializes all these information in the patient’s records. Disclosing them would be the equivalent of compelling the physician to testify on privileged matters he gained while dealing with the patient, without the latter’s prior consent. PRIEST-CONFESSANT PRIVILEGE A minister or priest cannot, without the confessant’s consent, be examined as to any confession made to or advice given by him in his professional character in the course of the discipline enjoined by the church to which the minister or priest belongs. - The communication must be made with intent to obtain penance and not merely for religious and spiritual counseling. - The confession and the advice must be in the course of the discipline enjoined by the church to which the confessant and the priest belongs. (2 Regalado 468). A “cross-over” confession is not included in the privilege. PUBLIC INTEREST PRIVILEGE A public officer cannot be examined during his term of office or afterwards, as to communications made to him in official confidence, when the court finds that the public interest would suffer by the disclosure. (S24(e) R130). Dean Riano opines that this privilege may be invoked as to informers. Banco Filipino v. Monetary Board, 142 S 523. Tapes and transcripts on MB closure of Banco Filipino not absolutely confidential. No showing that public interest would suffer. Danger of bank run absent since BF already closed. Executive Privilege Q What is executive privilege? A It is the privilege which protects the confidentiality of conversations that take place in the President’s performance of his official duties. The privilege may be invoked not only by the President but also by his close advisors under the “operational proximity” test. (Neri v. Senate Committee on Accountability of Public Officers and Investigations, G.R. No. 180643, 25 March 2008) The Supreme Court upheld Mr. Neri’s invocation of executive privilege (more specifically the presidential communications privilege) stating that disclosure might impair our diplomatic as well as economic relations with China. The SC distinguished this from U.S. v. Nixon, where there was an on-going criminal investigation into the Watergate break-in. (Neri v. Senate Committee on Accountability of Public Officers and Investigations, G.R. No. 180643, 25 March 2008) Trade & Industrial Secrets Privilege

Jurists Bar Review Pointers on Evidence 2015. All rights reserved 2015 by Jurists Review Center, Inc. Unauthorized reproduction, use, or dissemination strictly prohibited and shall be prosecuted to the full extent of the law, including administrative complaints with the Office of the Bar Confidant Supreme Court. Bar Review Pointers on Evidence by Prof. Manuel R. Riguera for Jurists Bar Review Center

9

Trade and industrial secrets recognized under our law and exempted from discovery and compulsory disclosure. (Air Philippines Corp. v. Pennswell, Inc., 13 December 2007). FILIAL PRIVILEGE A person may not be compelled to testify against his ascendants or descendants. (S25 R130). Note that the privilege belongs to the witness not to the person he is testifying against. A stepmother may be compelled to testify against her stepdaughter. The filial privilege does not apply since the same covers only direct ascendants and direct descendants, and a stepdaughter is not a direct descendant of the stepmother. (Lee v. Court of Appeals, 13 July 2010). PARTY ADMISSIONS Admission. An act, declaration, or omission of a party as to a relevant fact may be given in evidence against him. An admission need not be against the party’s interest. The admissions under R130 are extrajudicial admissions. This should be distinguished from judicial admissions under R129. Distinctions: 1. AS TO CONCLUSIVE EFFECT. 2. AS FOR NEED OF PROOF. Admissions may be made by a party directly or vicariously. Under the rule of res inter alios acta, the admissions of another person should not be considered as the admissions of a party. Exception is in the case of vicarious admissions. Rationale: Sufficient affinity or close relation of the 3P with the party litigant. 3 FORMS OF PARTY ADMISSION (ASO) -

Act Statement or declaration. Omission.

-

Voluntary participation in the re-enactment of a crime conducted by the police. Flight from the scene of a crime. Changing one’s appearance or name. Transfer of property. Attempt to bribe witnesses.

Act

Certain conduct on public-policy grounds not admissible as admission of liability. -

Post-accident repairs. Withdrawn or unaccepted offers of plea of guilty. Offers to pay or the payment of medical expenses. Offers of compromise in civil cases.

In criminal cases, offer of compromise implied admission of guilt. Exceptions: Quasi-offenses and those allowed by law to be compromised. Offer of accused to marry rape victim was held to be an implied admission. Accused’s silence when his wife’s nephew asked him why he killed his wife is an admission by silence under S32 R130. In addition, accused’s act of pleading for his sister-in-law’s forgiveness may be

Jurists Bar Review Pointers on Evidence 2015. All rights reserved 2015 by Jurists Review Center, Inc. Unauthorized reproduction, use, or dissemination strictly prohibited and shall be prosecuted to the full extent of the law, including administrative complaints with the Office of the Bar Confidant Supreme Court. Bar Review Pointers on Evidence by Prof. Manuel R. Riguera for Jurists Bar Review Center

10

considered as analogous to an attempt to compromise, which can be received as an implied admission of guilt under S27 R130. (Pp v. Español, 13 Feb 09). The offer of compromise dated 5 December 2000 was made prior to the filing of the criminal complaint against her on 9 March 2001 for a violation of the Bouncing Checks Law. The Offer of Compromise was clearly not made in the context of a criminal proceeding and, therefore, cannot be considered as an implied admission of guilt. (San Miguel Corp. v. Kalalo, 13 June 2012). Failure to respond to a demand letter is not an implied admission of liability. A person does not make a letter evidence by sending it to a party against whom he wishes to prove the facts stated therein. He can no more impose a duty to answer a charge than he can impose a duty to pay by sending goods. (Phil. First Ins. Co. v. Wallem Phils., 26 March 2009). Non-flight is not evidence of innocence. (Eduarte v. People, 16 April 2009). VICARIOUS ADMISSIONS (J-CAPP) AGENT & PARTNER The act or declaration of a partner or agent of the party within the scope of his authority and during the existence of the partnership or agency may be given in evidence against such party after the partnership or agency is shown by evidence other than such act or declaration. Estrada v. Desierto. Angara as Ex Sec was the alter-ego of Erap. JOINT OWNER, DEBTOR, INTEREST The act or declaration of a joint owner, joint debtor, or other person jointly interested with the party within the scope of his authority and during the existence of the joint ownership, debt, or interest may be given in evidence against such party after the joint ownership, debt, or interest is shown by evidence other than such act or declaration. Joint here should not be confused with joint cf solidary in our law of oblicon. It means community of interest. Thus a co-owner, solidary (not joint) debtor, and sureties/guarantors. The statements of Ayala Land Inc., (the developer) would be binding upon the Ayala Alabang Village Association (the homeowner’s association) in an action by the latter to enforce the deed of restrictions over a lot in Ayala Alabang. This is because ALI under the deed of restrictions may enforce the same together with AAVA and thus ALI is a person jointly interested under S28 R130. (The Learning Child Inc. v. Ayala Alabang Village Association, 7 July 2010). Q MBMI a Canadian corporation, entered into a joint venture with corporations A, B, and C. Redmont Corporation filed a petition to cancel the mining licenses of A, B, and C corporations on the ground that the foreign equity exceeded 40%. The Court of Appeals in finding on appeal that MBMI owned more than 60% of the capital stock of corporations A, B, and C, took into account the statements of MBMI to that effect. Corporations A, B, and C argued that MBMI’s statements are res inter alios acta since MBMI is not a party to the case. Is this argument correct? A No. The CA correctly applied S29 R130 on joint interest. By entering into a joint venture, MBMI has a joint interest with Corporations A, B, and C. Hence its statements in relation to such joint venture are an exception to the res inter alios acta rule. (Narra Nickel Mining Corp. v. Redmont Consolidated Mines Corp., 21 April 2014).

PRIVY

Jurists Bar Review Pointers on Evidence 2015. All rights reserved 2015 by Jurists Review Center, Inc. Unauthorized reproduction, use, or dissemination strictly prohibited and shall be prosecuted to the full extent of the law, including administrative complaints with the Office of the Bar Confidant Supreme Court. Bar Review Pointers on Evidence by Prof. Manuel R. Riguera for Jurists Bar Review Center

11

Where one derives title to the property from another, the act, declaration, or omission of the latter, while holding the title, in relation to the property, is evidence against the former. Privy: Per the rules a person from whom one derives the title to property from. E.g. donor, vendor, decedent, testator. CO-CONSPIRATORS Refers only to EJ acts or admissions and not to testimony given in court by the conspirator where there is a right to cross the conspirator. Take note of the common requirements that (1) the act or declaration must be in connection with the third person’s relation with the party, (2) the act or declaration must be made during the existence of the relation, and (3) the relation must be proved by evidence other than the act or declaration. In the case of admission by privies, however, independent evidence is not required. No. 2 is important and the subject of several bar exam questions. Confession An extrajudicial confession is binding only upon the confessant, even if it implicates others in an alleged conspiracy if there is no proof of the conspiracy aside from the extrajudicial confession. (Tamargo v. Awingan, 19 January 2010). Accused’s confession to a radio broadcaster was admissible in evidence. He was not entitled to the Miranda rights as he was not under custodial investigation. (People v. Hipona, 18 February 2010). Accused’s confession to a Bantay Bayan made without the assistance of counsel is inadmissible in evidence. Inquiry made by a bantay bayan has the color of a state-related function and entitles the suspect to his Miranda rights. (People v. Lauga, 15 March 2010). On the other hand, the accused’s admissions before barangay tanod and barangay officials made in public with other persons present were held admissible notwithstanding that the accused was not afforded his Miranda rights. (People v. Sace, 5 April 2010). ADOPTIVE ADMISSION: Adoptive admission is where a party, by his words or conduct, voluntarily adopts or ratifies another’s statement. Evidence of the statement would then be admissible against the party. Here the Respondent adopted its counsel’s statement that he had transmitted the draft of the answer to the Respondent, but did not sign the answer which was filed. (Republic of the Philippines v. Kenrick Dev’t Corp., G.R. 149576, 8 August 2006). SIMILAR ACTS RULE (RIAA 2nd branch) Evidence that one did or did not do a certain thing at one time is not admissible to prove that he did or did not do the same or similar thing at another time; but it may be received to prove PIKICHUS. Exception: PIKICHUS Plan, identity, knowledge, intent, custom, habit, usage, system, and the like. The 3 contracts to sell were not sufficient to prove a habit or custom on the part of XEI to grant the 120month term to all its lot buyers. There was no adequacy of sampling and uniformity of response. (Boston Bank v. Manalo, 9 Feb 06). Sexual abuse shield rule In criminal cases involving child abuse, evidence of the alleged child victim’s other sexual behavior and sexual predisposition (osb) is inadmissible. (S30 RECW).

Jurists Bar Review Pointers on Evidence 2015. All rights reserved 2015 by Jurists Review Center, Inc. Unauthorized reproduction, use, or dissemination strictly prohibited and shall be prosecuted to the full extent of the law, including administrative complaints with the Office of the Bar Confidant Supreme Court. Bar Review Pointers on Evidence by Prof. Manuel R. Riguera for Jurists Bar Review Center

12

Exc: To prove that a person other than the accused was the source of the semen, physical injury or other physical evidence (hair, blood). (SPie) The proponent must file a pretrial motion stating that he intends to offer such kind of evidence, in order to prevent unfair surprise upon the other. Rape shield rule under S6 of RA 8505 (Rape Victim Protection & Assistance Act) Evidence of victim’s past sexual conduct, opinion thereof, or of his/her reputation (PaSCOR) inadmissible unless court finds that such evidence is relevant to the case. HEARSAY RULE AND ITS EXCEPTIONS HEARSAY: An out-of-court statement which is relevant for the truth of a matter asserted therein. (See Federal Rules of Evidence). Self-serving statements are those made by a party out of court advocating his own interest. They do not include a party’s testimony in court as a witness. The proper ground for objecting to self-serving statements is not that they are self-serving but that they are hearsay. Statements made by a party in court favoring his interest may not be objected to as self-serving since the party may be cross-examined. (People v. Omictin, 26 July 2010). DETECTING HEARSAY AND DETERMINING ITS ADMISSIBILITY 1. The first step is to look out for the declarant. A declarant is a person who makes or writes an out-ofcourt statement. Easier to spot is oral hearsay where a witness will testify about what another person (the declarant) said out-of-court. But what should also be on the look out for written hearsay. If the witness will present or identify a document or letter written not by the witness himself but by another person who is not presented in court, a hearsay situation arises. The third class of possible hearsay is reputation. Reputation is hearsay since it is the collective statement of a community or group of persons who themselves are not presented in court. RATIONALE: -

(COD) The lack of opportunity to cross-examine the out-of-court declarant. The declarant’s statement is not under oath. No opportunity to observe the demeanor of the declarant.

2. The next step is to determine the purpose for which the declaration or statement is offered. If it is offered not to prove the truth of a matter asserted therein, then the declaration or statement is not hearsay and is admissible. INDEPEDENTLY RELEVANT STATEMENT. An out-of-court statement which is relevant not for the truth of a matter asserted therein but for something else, e.g., legal effect, mere fact of utterance, belief, intent, or state of mind. (LUBIS) A declaration by the deceased testator that there were Martians in his backyard is admissible in the probate of the testator’s will in order to prove that the testator was not of sound and disposing mind. In an action for recovery of land in which there were improvements constructed by the defendant’s deceased father, the father’s declaration that he owned the land is relevant not for its truth but to show that the father was a builder in good faith. Note that the statement cannot be treated as a vicarious admission by a privy since it is not against the father’s interest. State Of Mind Statements regarding the declarant’s then state of mind or intention are admissible as an exception to the hearsay rule. (FRE 803[3]). Statements of a declarant from which an inference may be made as to the state of mind of another, that is, the knowledge, belief, motive, good or bad faith, etc. of the latter, are

Jurists Bar Review Pointers on Evidence 2015. All rights reserved 2015 by Jurists Review Center, Inc. Unauthorized reproduction, use, or dissemination strictly prohibited and shall be prosecuted to the full extent of the law, including administrative complaints with the Office of the Bar Confidant Supreme Court. Bar Review Pointers on Evidence by Prof. Manuel R. Riguera for Jurists Bar Review Center

13

also not covered by the hearsay rule. (Estrada v. Desierto). The state of mind rule was recognized by the Supreme Court in the case of Estrada v. Desierto, where the court held that Angara’s diary containing statements regarding Erap’s intent to resign was admissible. The SC’s approach was to treat statements on state of mind as non-hearsay rather than an exception to the hearsay rule. Estrada v. Desierto, G.R. 146710-15, 3 April 2001: The Angara Diary contains statements of Erap which reflect his state of mind and are circumstantial evidence of his intent to resign. It also contains statements of Secretary Angara from which one can reasonably infer Erap’s intent to resign. Such statements are independently relevant and are excluded from the hearsay rule. 3. The last step is to determine if the hearsay statement or declaration falls within any of the exceptions to the hearsay rule, in which case the statement or declaration although hearsay is admissible. The newspaper clipping of a report is hearsay since the reporter who wrote the news account was not presented in court and hence could not be cross-examined by the accused as to the accuracy or veracity of his report. In an action to recover necessary and useful expenses, statement by decedent that he was the owner of the land admissible to prove his belief, that he was the owner and thus a possessor in good faith. In a marriage nullity case, the lack of personal interview of the respondent does not render hearsay the psychologist’s report. (Camacho-Reyes v. Reyes, G.R. No. 185286, 18 August 2010). Even hearsay evidence can be admitted if it satisfies the basic minimum test of relevance and consistency with other evidence. The courts should exercise of flexibility in the consideration of evidence, including hearsay evidence, in extrajudicial killings and enforced disappearance cases. (Razon v. Tagitis, 3 December 2009, Brion, J.) NBI agent’s testimony that Zaldy had identified in a police line-up the accused as the perpetrators of the robbery and killing was held unreliable where Zaldy did not testify in court and the NBI agent did not state when the line-up took place; how this line-up had been conducted; who were the persons in the line-up with the accused (if there were indeed other persons included in the line-up); and whether the line-up was confined to persons of the same height and built as the accused. (Pp v Cachuela, 10 June 2013). It should also be noted that the NBI agent’s testimony on Zaldy’s out-of-court identification is hearsay. The affiant’s failure to identify their affidavits in the preliminary investigation before the OMB and the grave nature of the charges (grave misconduct) led the SC to treat their affidavits as inadmissible under the hearsay rule. (Miro v. Vda de Erederos, 20 Nov 2013). Medical certificate that plaintiff suffered whiplash is hearsay where the physician who executed it not presented in court. (Dela Llana v Biong, 4 Dec 2013). Q Prosecution for rape. Dr. Mijares examined the victim AAA and executed a medical report on his findings. At the scheduled hearing, Dr. Mijares appeared, after several subpoenas and warnings from the court, but instead of presenting him to be examined on his medical report on the alleged rape of AAA, the prosecutor manifested that she was dispensing with his testimony provided the defense agreed to the prosecution’s offer of stipulation that AAA submitted herself to medical examination one week after the alleged rape, to which the defense acceded. Is the medical report hearsay? A Yes, since Dr. Mijares did not testify in court regarding the same. In fact his testimony was dispensed with. (People v. Rondina, 30 June 2014). (Note that the stipulation was not on the authenticity of the medical report but on the fact simply that AAA submitted herself to medical examination one week after the alleged rape). EXCEPTIONS

Jurists Bar Review Pointers on Evidence 2015. All rights reserved 2015 by Jurists Review Center, Inc. Unauthorized reproduction, use, or dissemination strictly prohibited and shall be prosecuted to the full extent of the law, including administrative complaints with the Office of the Bar Confidant Supreme Court. Bar Review Pointers on Evidence by Prof. Manuel R. Riguera for Jurists Bar Review Center

14

Exceptions may be grouped into two: Those where there is a requirement that the declarant is dead or unavailable to testify (DU requirement) and those where there is no such requirement. Unavailability. Means that the declarant cannot be produced in court by a party despite diligent efforts, as when he could no longer be located. Mere refusal to testify is not equivalent to unavailability as declarant can be subpoenaed, unless declarant is invoking a privilege. DEATH/UNAVAILABILITY OF DECLARANT REQUIRED (D BF PDC) DYING DECLARATION -

Requirements: DICC Applies to both civil and criminal cases Declarant should die Death is the subject of inquiry in the case Declarant conscious of impending death Declaration relates to cause and circumstances sorrounding declarant’s death. Declarant must otherwise be competent. Thus if the declarant said “Juan said it was Pedro who shot me,” the declaration is inadmissible for being hearsay. It does not matter whether the DD is against or in favor of the accused or adverse party. A statement which is not admissible as a dying declaration may be admitted as part of the res gestae. (Multiple admissibility). The dying declaration may be reduced into writing by a witness, in which case the writing is subject to the best evidence rule. If the one who wrote down the declaration is a police officer and the recording was made in a police investigation report, two levels of hearsay are involved: official entries and dying declaration/res gestae. The police investigation report may be presented even without presenting the police officer who made it.

Under the Rules on Evidence, a dying declaration is an exception to the hearsay rule and admissible in evidence. Emong gave the statement under a consciousness of an impending death. Even if he did not make an explicit statement of that realization, the degree and seriousness of his wounds and his death shortly after sustaining the wounds is sufficient to show that Emong was conscious of his dying condition. (Marturillas v. People, G.R. 163217, 18 Apr 06). The fact that the victim had nine stab wounds which caused his death within the next 48 hours indicates that the victim was conscious of his impending death. (People v. Tabarnero, 24 February 2010). Bloodied condition of a stab victim indicates that victim must have been fully aware that he was on the brink of death. (People v. Serenas, 29 June 2010). It is of no moment that the victim died seven days from the stabbing incident and after receiving adequate care and treatment, because the apparent proximate cause of his death, the punctures in his lungs, was a consequence of appellant’s stabbing him in the chest. At the time the victim made his declaration, his breathing labored, he realized that he could die after having been stabbed twice in the chest. (People v. Rarugal, 16 January 2013). BUSINESS ENTRIES (S43 R130) -

-

Requirements: TDK PO Entries made at, or near the time of the transactions to which they refer, by a person deceased, or unable to testify, who was in a position to know the facts stated therein. The entrant must have made the entries in his professional capacity or in the performance of duty and in the ordinary or regular course of business or duty. (PDOR) The use of the word “business” is somewhat misleading. Not limited to “business” in the ordinary sense of the word but extends to all cases were the entries were made in a professional capacity or in the performance of a duty. Even a housewife’s budget journal is a business record. Maybe legal or illegal. Thus the diary of a jueteng bookie where he

Jurists Bar Review Pointers on Evidence 2015. All rights reserved 2015 by Jurists Review Center, Inc. Unauthorized reproduction, use, or dissemination strictly prohibited and shall be prosecuted to the full extent of the law, including administrative complaints with the Office of the Bar Confidant Supreme Court. Bar Review Pointers on Evidence by Prof. Manuel R. Riguera for Jurists Bar Review Center

15

-

kept records of bets and winnings is a business record and admissible to prove the contents thereof. Examples: Books of account, invoices, statements of account, bills of lading, log-book of a security guard, medical charts by nurses, interns or resident physicians in a hospital. The entrant should have personal knowledge of the information he is entering. Canque v. CA, 305 SCRA 579. Entries made by a bookkeeper in her book of collectible accounts hearsay since she did not have personal knowledge thereof. Compare with the electronic business records exception under S1 R8 REE. With respect to the identification of the sales invoices, Haw’s testimony was hearsay because he was not present during its preparation and the secretaries who prepared them were not presented to identify them in court. Further, these sales invoices do not fall within the exceptions to the hearsay rule even under the “entries in the course of business” because the petitioners failed to show that the entrant was deceased or was unable to testify. (Advance Paper Corp. v. Arma Traders Corp., 11 Dec 2013).

ELECTRONIC BUSINESS RECORD (S1 R8 REE) -

Requirements: MEAT KKR A memorandum, report, record or data compilation (MR RD) of acts, conditions, events, diagnoses, or opinions (ACEDO) made by electronic, optical or other similar means at or near the time by, or from transmission or supply of information by, a person with knowledge thereof, kept in the regular course or conduct of a business activity, (rc-cba) and such was the regular practice to make the MR RD by electronic, optical or similar means

is excepted from the rule on hearsay evidence. (S1 R8 REE). The above matters should be shown by the testimony of the custodian or other qualified witness. Such testimony may be in affidavit form as per S1 R9 REE. Sec. 2(b) R2 of REE defines “business records” as including records of any business, profession, occupation, and calling of every kind, whether or not conducted for profit or for legitimate purposes. Distinctions Between Business Entries Exception (BEE) & The Electronic Business Record Exception (EBRE) Under BEE, it is required that the entrant be dead or unavailable to testify, while there is no such requirement under the EBRE. Under BEE, the entrant should be in a position to know the facts stated therein, while in EBRE, the entrant or recorder need not personally know the facts entered, it being sufficient that the records were made “from transmission or supply of information by a person with knowledge thereof.” FORMER TESTIMONY OR DEPOSITION (S47) Requirements: PSC The testimony or deposition of a witness deceased or unable to testify, given in a former case or proceeding, judicial or administrative, involving the same parties and subject matter, may be given in evidence against the adverse party who had the opportunity to cross-examine the witness. (PSC) Not only the testimony or deposition, but the documents which are part of the testimony or deposition, are admissible under this exception. (Manliclic v. Calaunan, 25 January 2007).

Jurists Bar Review Pointers on Evidence 2015. All rights reserved 2015 by Jurists Review Center, Inc. Unauthorized reproduction, use, or dissemination strictly prohibited and shall be prosecuted to the full extent of the law, including administrative complaints with the Office of the Bar Confidant Supreme Court. Bar Review Pointers on Evidence by Prof. Manuel R. Riguera for Jurists Bar Review Center

16

Statements made during preliminary investigation are not covered by this exception since there is no right to cross-examine. (S3(e) R112). But depositions are covered under this exception because the deponents may be cross-examined. (S3 R23, S4 R24). S47 R130 refers to a deposition taken in a former case or proceeding. If the deposition is taken in the same case or proceeding, it is S4(c) R23 (DOSUE) which governs. Francisco v. People, GR 146584, 12 July 2004, Callejo, J.: Jovita’s testimony in Case 1 (qualified theft) that Pacita confessed to her that she had sold the jewelry to Francisco is inadmissible against Francisco in Case 2 (fencing) to prove the truth of said admission. It bears stressing that Francisco was not a party (accused) in Case 1. Jovita did not reiterate her testimony in Case 2 nor was Pacita presented as a witness therein to testify on the admission she purportedly made to Jovita. Thus there was no opportunity to cross-examine Pacita. Pacita’s testimony in the preliminary investigation of Case 2 as well as her affidavit are inadmissible against Francisco since the latter did not have the opportunity to crossexamine Pacita. The testimony of witnesses in a prior criminal case for reckless imprudence against the employee (bus driver) cannot be introduced in a subsequent civil case for tort filed by the offended party against the employer (bus company) to enforce the latter’s vicarious liability under Article 2180 of the Civil Code. This is because the employer is not a party in the criminal case and thus had no opportunity to crossexamine the witnesses. However where the employer failed to object to the TSNs when they were offered in evidence, the same are admissible. The driver’s acquittal is of no moment since the tort case is an independent civil action. (Manliclic v. Calaunan, 25 January 2007). A judgment of conviction of the employee is however conclusive upon the employer in a motion or action to enforce the employer’s subsidiary liability under Article 103 of the Revised Penal Code and said judgment is admissible in evidence in the proceedings brought to enforce the employer’s subsidiary liability. ACT OR DECLARATION ABOUT PEDIGREE -

-

-

-

Requirements: DRAPE The act or declaration of a person deceased or unable to testify in respect of the pedigree of another person related to him by birth or marriage, may be received in evidence where it occurred before the controversy (ante litem motam), and the relationship between the two persons is shown by evidence other than such act or declaration. The declaration must be about the pedigree of a relative, that is, a person related to the declarant by birth or marriage. E.g., Declarant says that R (his relative) has an illegitimate son named Boris. The declarant’s relation by birth or marriage to the relative must be preliminarily proved by independent evidence other than the declaration. Thus in the prior example, the proponent must first show the relationship between declarant and R before the declarant’s statement can be admitted in court. However if the declarant’s statement is about his relationship to R, then preliminary proof of relationship is no longer required. Thus if the declarant’s statement was that he is the illegitimate son of R, then preliminary proof is no longer required. (See Tison v. CA, 31 July 1997).

DECLARATION AGAINST INTEREST (S38) -

-

Requirements: D ITC The declaration made by a person deceased or unable to testify, against the interest of the declarant, if the fact asserted in the declaration was at the time it was made so far contrary to the declarant’s own interest, that a reasonable man in his position would not have made the declaration unless he believed it to be true, may be received in evidence against himself or his successors in interest and against third persons. Against interest means against the declarant’s pecuniary, moral, and penal interest. A declaration which would render invalid a claim by the declarant against another person is considered as against the declarant’s interest. (FRE 804[b][3]).

Jurists Bar Review Pointers on Evidence 2015. All rights reserved 2015 by Jurists Review Center, Inc. Unauthorized reproduction, use, or dissemination strictly prohibited and shall be prosecuted to the full extent of the law, including administrative complaints with the Office of the Bar Confidant Supreme Court. Bar Review Pointers on Evidence by Prof. Manuel R. Riguera for Jurists Bar Review Center

17

-

-

People v. Toledo, 51 Phil 285: The deceased declarant had stated that he alone was the one responsible for the death of the victim and that the accused was free from blame. While statement not admissible as dying declaration, it was admissible as a declaration against interest. Fuentes v. CA, 253 SCRA 430 involved a similar declaration but the accused did not exert diligent efforts to locate the declarant who was not shown to be dead. SC held that the declarant was not unavailable to testify and thus the exception did not apply.

There is no declaration against interest where the affidavit was executed not by the seller himself but by his father. (Dantis v. Maghinang, 10 April 2013). CHILD-DECLARANT RE CHILD ABUSE (S28 RECW) -

REQUIREMENTS: DN CCC Where the declarant is a child-witness, who is dead or unavailable to testify, and the declaration relates to act or attempted act of child abuse. Proponent must give advance notice to adverse party and give the particulars of the declaration. “Unavailable” includes cases where the child is suffering from physical infirmity, lack of memory, mental illness, or will be exposed to severe psychological injury. The declaration is corroborated by other admissible evidence.

DEATH/UNAVAILABILITY OF DECLARANT NOT REQUIRED (RFC COLES) RES GESTAE, PART OF THE (S42) -

Actually consists of two exceptions, excited utterances and verbal acts.

-

Excited utterance/spontaneous exclamation Statements made by a person while a startling occurrence is taking place or immediately prior or subsequent thereto with respect to the circumstances thereof (STIC) may be given in evidence as part of the res gestae. Excited utterance must be made while under the influence of a startling occurrence and at or about the time of the startling occurrence, and regarding the circumstances thereof. Rationale: A high level of reliability on statements made under the influence of a startling occurrence. The statement becomes part of the res gestae. It is the event speaking through the person rather than a person speaking about an event. Air France v. Carrascoso, 18 SCRA 155: airplane purser’s notation in his notebook re the commotion involving the forcible transfer of Carrascoso from first class to tourist class. In an American case, a woman was gravely injured after a mugging and lapsed into coma. She was brought into the hospital where she later regained consciousness. A police investigator brought pictures of suspects to her. When one picture was shown to her, the woman became extremely agitated and said, “He’s the one who mugged me.” The woman later died of her injuries. The court held that the testimony of the police investigator on what the woman said was admissible as an excited utterance. The showing of the picture to her was a startling occurrence.

-

-

Statement made as part of the res gestae admissible even if made eleven hours after the incident, provided the declarant was still under the influence of the startling event. (Zarate v. People, 3 July 2009). Verbal acts. -

Requirement: MEL Statements accompanying an equivocal act material to the issue and giving it a legal significance.

Jurists Bar Review Pointers on Evidence 2015. All rights reserved 2015 by Jurists Review Center, Inc. Unauthorized reproduction, use, or dissemination strictly prohibited and shall be prosecuted to the full extent of the law, including administrative complaints with the Office of the Bar Confidant Supreme Court. Bar Review Pointers on Evidence by Prof. Manuel R. Riguera for Jurists Bar Review Center

18

-

-

Reason for exception. The truth of the verbal act is not relevant, what is important is that the statement gives legal significance to the equivocal act. Strictly speaking, verbal acts are not an exception to the hearsay rule but are non-hearsay statements. Example: Witness for the estate stated that when the decedent handed the money to the defendant, the decedent said, “This is a loan.”

COMMON REPUTATION (S41) -

-

-

Requirements: PMMA Common reputation regarding: 1) facts of public or general interest more than 30 years old, 2) marriage, or 3) moral character. The reputation must exist ante litem motam. Note that the qualifying phrase, “more than 30 years old” only qualifies facts of public or general interest. Thus common reputation regarding marriage or moral character need not be more than 30 years old. The reason is that there is greater community interest in facts respecting marriage and moral character. Nature of the reputation evidence. S41 speaks of common reputation which is generally understood as the reputation existing in the community in which the fact to be proved is known or exists. (2 Regalado 494, 6th rev. ed.). As to reputation regarding moral character, it is submitted that the reputation may be not only among the community but also among his associates. (FRE 803[20]). The reason is that in modern urban communities, individuals are more anonymous. (ANTONIO R. BAUTISTA, BASIC EVIDENCE 139 [2004 ed.]). Monuments and inscriptions in public places may be received as evidence of common reputation. (S41 R130).

FAMILY REPUTATION OR TRADITION ABOUT PEDIGREE Requirements: AFP A family member’s testimony on the family reputation or tradition regarding the pedigree of any of its members. The reputation or tradition must exist prior to the controversy. (Ante litem motam). Ex.: W can testify that the reputation in their family was that Maria was an adopted child of his parents. Note that the witness must be a member of the family on whose reputation or tradition he is testifying about. Preliminary and independent evidence of the witness’s family membership not required. He can rely on his own testimony. Entries or engravings in family books or objects admissible as proof of pedigree without calling to testify the one who made the entries or engravings. COMMERCIAL LISTS (S45) -

-

-

Requirements: IPO PR Statements of matters of interest to persons engaged in an occupation contained in a list, register, periodical or other published compilation (LR PP), published for use by persons engaged in that occupation and generally used and relied upon by them. Examples: Stock market reports, foreign exchange tables, mortality tables, actuarial and annuity tables, which are published for use and relied upon by the persons engaged in the enterprise or business. Advertisements in the buy-and-sell section of newspapers not included since they are just separate and distinct offers. PNOC Shipping Corp. v. CA, 297 SCRA 402 (1998). Price quotation letters for the replacement cost of a barge sent by shipbuilders to PNOC are not covered by the exception.

Jurists Bar Review Pointers on Evidence 2015. All rights reserved 2015 by Jurists Review Center, Inc. Unauthorized reproduction, use, or dissemination strictly prohibited and shall be prosecuted to the full extent of the law, including administrative complaints with the Office of the Bar Confidant Supreme Court. Bar Review Pointers on Evidence by Prof. Manuel R. Riguera for Jurists Bar Review Center

19

ENTRIES IN OFFICIAL RECORDS (Official Entries) -

-

-

-

-

-

Requirements: OPP Entries in official records made in the performance of his duty by a public officer of the Philippines, or by a person in the performance of a duty specially enjoined by law. Entries must be made by a public officer or a private person specially enjoined by law to make such entries. Examples of private person specially enjoined by law to make entries: (1) Log book entry required under the Code of Commerce to be kept by a ship captain; (2) stamp of dishonor and its reason required from the drawee of a check under B.P. Blg. 22; (3) Parties to a marriage ceremony and the solemnizing officer are required to sign and attest the marriage certificate and the solemnizing officer is required to keep a copy. (Art. 23, Family Code). Is the medical certificate issued by a doctor in a private hospital admissible to prove the contents thereof without producing the doctor who made the entries? No. The medical certificate would not qualify as an official entry since the doctor is neither a public officer nor one specially enjoined by law to make entries. (ALBANO, REMEDIAL LAW REVIEWER 1294 [2007 ed.] citing Andrade v. Lao, CA-G.R. No. 21279, 29 June 1992). Nor would the medical certificate qualify as a business entry exception since the entrant is not dead or unavailable to testify. Entrant need not be presented in court to testify on the official entries. The entrant must have personal knowledge of the facts he is entering or the information must have been supplied by persons having a specific legal (not moral) duty to supply such information. (Africa v. Caltex, 16 SCRA 448). Thus in Barcelon, Roxas Securities Inc. v. Comm. of Internal Revenue, 7 Aug 06, a BIR record book containing the taxpayer’s name, the tax assessed, the registry receipt number, and the date of mailing, which was offered to prove that the taxpayer had received an assessment notice was held to be hearsay. The CTA found that the BIR records custodian who made the entries thereon did not have personal knowledge thereof since she did not attest that she was the one who personally prepared and mailed the assessment notice nor did she attest that the information in the entries were supplied by persons having a specific legal duty to do so. Africa v. Caltex, 16 SCRA 448. Police investigation report of a fire at a gasoline station. Police officer did not have personal knowledge of the cause of fire (throwing of a cigarette) which was supplied to him by a bystander he had interviewed. Sheriff’s return is an official entry as to fact stated therein. Sheriff need not be presented in court. (Manalo v. Robles Trans. Co., 99 Phil. 729). POEA certification that accused was not a licensed recruiter is an official record. Hence it is an exception to the hearsay rule and admissible even if the one who made the certification was not presented in court. (People v. Ochoa, 31 August 2011). The SC noted that a witness had identified the signature of the certifying officer. It is opined that even if the certifying officer’s signature was not identified, the certification would still be admissible as it is a public document which does not need to be authenticated. The NBI/Progress report, having been submitted by the officials in the performance of their duties not on the basis of their own personal observation of the facts reported but merely on the basis of the complainants' affidavits, is hearsay, in fact double hearsay. Thus, the Deputy Ombudsman cannot rely on it. (Miro v Vda de Erederos, 20 Nov 2013).

LEARNED TREATISES (S46) Requirements: PAR A published treatise, periodical or pamphlet (TPP) on a subject of law, art, science, or history (LASH) is admissible as tending to prove the truth of a matter stated therein if the court takes judicial notice, or a witness expert in the subject testifies that the writer of the statement in the treatise, periodical or pamphlet is recognized in his profession or calling as an expert in the subject.

Jurists Bar Review Pointers on Evidence 2015. All rights reserved 2015 by Jurists Review Center, Inc. Unauthorized reproduction, use, or dissemination strictly prohibited and shall be prosecuted to the full extent of the law, including administrative complaints with the Office of the Bar Confidant Supreme Court. Bar Review Pointers on Evidence by Prof. Manuel R. Riguera for Jurists Bar Review Center

20

Foundational testimony of another expert that the published expert is recognized as such in his field. Court may however take judicial notice of such expertise. ELECTRONIC BUSINESS RECORD (see discussion after business entries exception) ADMISSION AGAINST INTEREST In the United States, there is a divergence of view on whether to treat admissions against interest as non-hearsay or as an exception to the hearsay rule. Our Rules of Evidence treat admission against interest as non-hearsay as evidenced by its non-inclusion in the hearsay exceptions. However, in Estrada v. Desierto, the SC said that “admissions are admissible even if they are hearsay,” in effect treating them as an exception to the hearsay rule. MULTIPLE (DOUBLE) HEARSAY Where a hearsay declaration is embedded in another with the result that there are two or more levels of hearsay. This frequently happens where a party seeks to introduce a written report by a declarant (first level) of another declarant’s out-of-court statement (second level). Under FRE 805, hearsay included within hearsay is not excluded under the hearsay rule if each level of hearsay conforms with an exception to the hearsay rule. Illustration: D2 was mortally injured. Conscious of his impending death, D2 was interviewed by a police officer D1 and D2 told D1 that X stabbed him several times. D1 wrote what D2 said in his police investigation report. In the prosecution of X for murder, may the prosecution present the police investigation to prove that X stabbed D2 without presenting D1? Yes. Each level of hearsay conforms with an exception to the hearsay rule. The first level is justified by the official entries exception while the second level is justified by the dying declarations exception. If the one who wrote down what D2 said was a private person who later became unavailable to testify, the writing would be inadmissible as the first level is not supported by a hearsay exception. Q D2 was found mortally injured by a security officer in a lobby of a hotel. Conscious of his impending death, D2 was interviewed by the security police officer D1 and D2 told D1 that X stabbed him several times. D1 wrote what D2 said in his log book. D1 later died in an accident. In the prosecution of X for murder, may the prosecution present the log book to prove that X stabbed D2? Yes. Each level of hearsay conforms with an exception to the hearsay rule. The first level is justified by the business entries exception while the second level is justified by the dying declarations exception. OPINION RULE G.R. The opinion of a witness is not admissible in evidence. EXCEPTIONS: - Expert opinion. - Lay opinion on HIS and short-hand opinions. EXPERT OPINION The opinion of a witness on a matter requiring special knowledge, skill, experience or training (special TEKS) which he is shown to possess, may be received in evidence. - Note that the witness need not have formal training or instruction on his field of expertise. - The witness should be qualified, that is, he should be shown to possess the special knowledge, skill, experience or training, before he is allowed to testify. However the objection should be timely raised, otherwise it is deemed waived. - Unlike the ordinary witness, the expert witness may testify based on hypothetical or assumed facts. Thus a clinical psychologist can testify regarding the psychological capacity of a person even if she was not able to personally interview the witness.

Jurists Bar Review Pointers on Evidence 2015. All rights reserved 2015 by Jurists Review Center, Inc. Unauthorized reproduction, use, or dissemination strictly prohibited and shall be prosecuted to the full extent of the law, including administrative complaints with the Office of the Bar Confidant Supreme Court. Bar Review Pointers on Evidence by Prof. Manuel R. Riguera for Jurists Bar Review Center

21

-

-

Re marriage nullity cases under Art. 36 of the FC. In Republic v. Molina, 268 SCRA 198 (1997), the SC laid down the guideline that the psychological incapacity of a party must be sufficiently proved by qualified psychiatrists or clinical psychologists. This requirement was relaxed in Marcos v. Marcos, G.R. 136490, 19 Oct 2000, wherein it was stated that actual medical examination of the person alleged to be psychologically incapacitated is not an indispensable requirement. Even assuming Dra. Dela Llana was a neurologist, her testimony that the car collision caused the whiplash injury cannot be given probative value as she was presented not as an expert witness but as an ordinary witness. (Dela Llana v. Biong, 4 December 2013).

LAY OPINION -

-

Lay or short-hand opinion is allowed on HIS: o Handwriting of which he is familiar. o Identity of a person whom the witness he adequately knows. o Sanity of a person of whom the witness is acquainted. Short-hand impressions: Impressions on the CABE (Condition, appearance, behavior, and emotion) of a person. Witnesses can also give opinions re physical dimensions (length, height, weight) and speed, and lighting conditions.

CHARACTER EVIDENCE Difference between character and reputation. Character is what you are, reputation is what the community knows you are. Reputation may be used to prove character. Thus under S41 R131 common reputation may be used to prove moral character as an exception to the hearsay rule. A person’s moral character is relevant where issues of good or bad faith, fraud, violence, and immorality are involved. Persons of good moral character do not usually deceive, physically hurt, or commit immoral acts. The rules may be outlined thus: PARTIES -

-

CRIMINAL CASES. Evidence of the accused’s good moral character is admissible if pertinent to the moral trait involved in the offense charged (pmtioc). The prosecution cannot adduce evidence of the accused’s bad moral character unless the defense opens the door for rebuttal by introducing evidence of the accused’s good moral character. CIVIL CASES. Evidence of moral character of a party is admissible only when pertinent to the issue of character involved in a case. (picic)

OFFENDED PARTY IN CRIMINAL CASE -

-

Evidence of the good or bad moral character of the offended party is admissible if it tends to establish the probability or improbability of the offense charged (tepioc). Thus: Evidence of the violent character of the victim is relevant to prove self-defense by the accused. Evidence of his non-violent character can be adduced by the prosecution to show the accused’s guilt. In rape cases, evidence of the victim’s promiscuous character may be adduced by the accused to show consent. The accused in a murder case raised the defense that victim was a drug addict and thief to try to show that he could have been killed by any one of those from whom he had stolen. The SC said that proof of the bad moral character of the victim is irrelevant to establish the probability or improbability of his killing since the accused did not allege that the victim was the aggressor or that the killing was made in self-defense. (Pp v. Lee, G.R. No. 139070, 29 May 2002).

WITNESSES

Jurists Bar Review Pointers on Evidence 2015. All rights reserved 2015 by Jurists Review Center, Inc. Unauthorized reproduction, use, or dissemination strictly prohibited and shall be prosecuted to the full extent of the law, including administrative complaints with the Office of the Bar Confidant Supreme Court. Bar Review Pointers on Evidence by Prof. Manuel R. Riguera for Jurists Bar Review Center

22

-

The adverse party may introduce evidence that a witness’s general reputation for honesty, integrity, or truth (HIT) is bad for purposes of impeaching the witness. (S11 R132). If impeached, the party presenting the witness may rehabilitate his witness by presenting evidence of his good moral character. (S14 R132).

How Character Proved: (1) Reputation evidence, (2) specific instances of conduct, or (3) the witness’ opinion. METHODS OF PROVING CHARACTER Reputation evidence Evidence of common reputation is the method of proving character. Under S41 R130 common reputation is admissible to prove moral character. Under S11 R132 a witness’s general reputation for honesty, integrity, or truth may be impeached by reputation evidence. Traditionally common reputation has been construed as that existing in the community in which a person lives or is well-known. (2 FLORENZ D. REGALADO, REMEDIAL LAW COMPENDIUM 494 [6th rev. ed.]). The modern view is that reputation among a person’s associates is also admissible to prove his character. (See FRE 803[21]). The basis for this view is that in urban societies, individuals tend to be more anonymous. (ANTONIO R. BAUTISTA, BASIC EVIDENCE 139 [2004 ed.]). Specific instances of conduct Specific instances of conduct may be used to prove character, where such character amounts to or is similar to a custom or habit. (S34 R130). The specific instances must be numerous enough to justify a finding that such past conduct amounts to a custom or habit. Note however that a witness’ general reputation for honesty, integrity, or truth may not be impeached by specific past acts of lying, except that the witness may be examined as to the fact of a final conviction. Witness’ opinion It appears under the Rules of Evidence that opinion is inadmissible to prove character as it is not among those excepted from the opinion rule. Under FRE 405(a) however, testimony in the form of an opinion is admissible to prove character. BURDEN OF PROOF Distinction between burden of proof and burden of evidence. Burden of proof is the duty of a party to present evidence on the facts in issue necessary to establish his claim or defense by the amount of evidence required by law. (S1 R131). The BOP rests upon a party asserting the affirmative of an issue. Thus Plaintiff who alleges defendant’s negligence has the burden of proof. This is often with the Plaintiff but it need not necessarily be so. The Defendant in a suit for enforcement of written contract who raises the defense of forgery has the burden of proof of proving forgery. The key query is who is the party asserting the affirmative of an issue. Burden of evidence on the other hand is the duty of a party to go forward with the evidence and may shift from party to party depending upon the developments in the case. Rivera v. CA, G.R. 115625, 23 Jan 1998 (Equipoise Doctrine). Where the evidence on an particular issue is in equipoise or level, the party with the burden of proof on that issue will lose out on that particular issue. PRESUMPTIONS

Jurists Bar Review Pointers on Evidence 2015. All rights reserved 2015 by Jurists Review Center, Inc. Unauthorized reproduction, use, or dissemination strictly prohibited and shall be prosecuted to the full extent of the law, including administrative complaints with the Office of the Bar Confidant Supreme Court. Bar Review Pointers on Evidence by Prof. Manuel R. Riguera for Jurists Bar Review Center

23

Most presumptions are based on logical inferences. An underlying fact gives a reasonable inference to prove that another fact (the presumption) exists, since there is a logical connection between the fact in existence and the fact proved. Essentially a presumption is based on a probability (albeit a high one), that is, if an underlying fact exists it is highly probable that the presumed fact also exists. Since it is based on a probability, a presumption may be rebutted. An example of this kind of presumption is that a person found in possession of a thing taken in the doing of a recent wrongful act is the taker and the doer of the whole act. (S3[i] R131). Other presumptions are based on the generally known results of wide human experience. An example is the presumption that a person takes ordinary care of his concerns. (S3[d] R131). Two kinds of presumptions: conclusive and disputable. A conclusive presumption is not based on a probability but on policy considerations. -

-

Presumption on estoppel. (S2[a] R131). Presumption barring tenant from denying landlord’s title. (S2[b] R131). The presumption under S2(b) R131 does not bar a tenant from challenging the landlord’s title by title subsequent to the commencement of the lease contract, as where the tenant had purchased the leased land from the mortgagee-purchaser during the extrajudicial foreclosure sale. (Ermitano v. Paglas, 23 January 2013). Presumption re negotiable instrument’s valid delivery and filling-in if in the hands of a HDC. (S14 & 16, NIL). Presumption re insurer’s written acknowledgment of premium payment insofar as to validate insurance contract. (S78 IC).

Salient disputable presumptions: - That evidence willfully suppressed would be adverse if produced. (S3[e]). The presumption does not apply from the prosecution’s failure to present the NBI agents and the fingerprint and paraffin tests, since the defense could have obtained the said evidence which was equally available to it. Also, if the suppression is in exercise of a privilege, like the attorney-client privilege, the presumption does not apply. - That there was a sufficient consideration for a contract. (S3[r]). - That a negotiable instrument was given or indorsed for a sufficient consideration. (S3[s]). In Travel-On v. CA, 210 SCRA 351, the SC held that a check is by itself sufficient evidence of the debt and that the check holder does not need to present the covering documents or invoices. - That a letter duly directed and mailed was received in the regular course of mail. S3(v). Hence one need not present the return card. The facts to be proved in order to raise this presumption are: (1) the letter was properly addressed with postage prepaid, and (2) the letter was mailed. (Barcelon, Roxas Securities v. CIR, G.R. 157064, 7 August 2006). Note however that in BP 22 cases, the SC requires actual proof that the accused received notice of dishonor. - That a man and a woman deporting themselves as husband and wife have entered into a lawful contract of marriage. (S3[aa]). Hence they need not present a marriage certificate. - That a printed or published book, purporting to be printed or published by a public authority, was so printed or published. (S3[gg]). - That a printed or published book, purporting to contain reports of cases adjudged in tribunals of a foreign country where the book is published, contains correct reports of such cases. (S3[hh]). Thus you do not need to get a certified true copy of the case. WHAT NEED NOT BE PROVED: JUDICIAL NOTICE AND ADMISSIONS (Rule 129) JUDICIAL NOTICE -

Mandatory:

(SIP NTG)

Jurists Bar Review Pointers on Evidence 2015. All rights reserved 2015 by Jurists Review Center, Inc. Unauthorized reproduction, use, or dissemination strictly prohibited and shall be prosecuted to the full extent of the law, including administrative complaints with the Office of the Bar Confidant Supreme Court. Bar Review Pointers on Evidence by Prof. Manuel R. Riguera for Jurists Bar Review Center

24

o o o o o o -

-

-

-

-

-

-

The existence and territorial extent of STATES, their political history, forms of government, and symbols of nationality. INTERNATIONAL LAW, the admiralty and maritime courts of the world and their seals. The constitution and history of the PHILIPPINES, the official acts of the legislative, executive, and judicial departments of the Philippines. The laws of NATURE. The measurement of TIME. The GEOGRAPHICAL divisions.

Discretionary (PUJ). Matters which are of PUBLIC knowledge, capable of UNQUESTIONABLE demonstration, or ought to be known to JUDGES because of their judicial functions. Hearing is necessary before Court can take judicial notice. During the trial, the court, on its own initiative, or on request of a party, may announce its intention to take judicial notice of any matter and allow the parties to be heard thereon. After the trial, and before judgment or on appeal, the proper court, on its own initiative or on a party’s request, may take judicial notice of any matter and allow the parties to be heard thereon if such matter is decisive of a material issue in the case. (S3 R129). Courts cannot take judicial notice of foreign laws. The parties may however stipulate on what the foreign law is in which case the same are judicial admissions binding upon them and they cannot take a contrary stance. (PCIB v. Escolin, G.R. 27936, 29 March 1974). Court of Appeals erred in taking judicial notice of the alleged practice of prospective lessees in the Baclaran area to pay goodwill money to the lessor. Requirement of notoriety not met. (Sps. Latip v. Chua, 16 October 2009). Courts cannot take “judicial knowledge that criminal activities such as robbery and kidnappings are becoming daily fares in Philippine society.” (New Sun Valley Homeowners’ Assoc. v. Sangguniang Bgy. of Sun Valley, 27 July 2011). The Management Contract entered into by Asian Terminals Inc. and the Philippine Ports Authority (PPA) cannot be considered an official act of the executive department. The PPA is a government-owned and controlled corporation in charge of administering the ports in the country. Obviously, the PPA was only performing a proprietary function when it entered into a Management Contract with ATI. As such, judicial notice cannot be applied. (Asian Terminals Inc. v. Malayan Insurance Co., 4 April 2011). Courts cannot take judicial notice that vehicular accidents cause whiplash injuries. (Dela Llana v. Biong, 4 Dec 2013).

JUDICIAL ADMISSIONS (S4 R129). Cannot be contradicted except if made through palpable mistake. a) Section 7, 8, 10, and 11 of Rule 8. (Actionable document and general denial). b) Section 8, Rule 10. (Admissions in superseded pleadings) Downgraded to extrajudicial admissions. (Ching v. CA, 331 SCRA 16). c) Section 2 & 3 Rule 26. (Failure to answer request for admission). Note that the admission under S2 R26 is for purposes of the pending action only. Failure to answer the complaint is not an implied admission of the allegations thereof. (Heirs of de Guzman v. Perona, 2 July 2010). The benefit of a judicial admission may be lost by failure to object to the presentation of evidence contradicting it. (ATCI Overseas Corp. v. Echin, 11 October 2010). PRESENTATION OF EVIDENCE EXAMINATION OF A WITNESS (S1) G.R. The examination of a witness shall be done in open court (oral examination). Exceptions:

Jurists Bar Review Pointers on Evidence 2015. All rights reserved 2015 by Jurists Review Center, Inc. Unauthorized reproduction, use, or dissemination strictly prohibited and shall be prosecuted to the full extent of the law, including administrative complaints with the Office of the Bar Confidant Supreme Court. Bar Review Pointers on Evidence by Prof. Manuel R. Riguera for Jurists Bar Review Center

25

-

-

S9 & 15 Rule on Summary Procedure. SC Guidelines on Pre-trial and Deposition. S1 R9 Rules on Electronic Evidence. Matters regarding the admissibility and evidentiary weight of electronic documents may be proved by affidavits subject to cross by adverse party. Cases covered by the Judicial Affidavit Rule

The witness’s examination should be done in question-and-answer form. Narrative testimony prohibited. Exception is S19, Rule on Examination of a Child Witness. S2 -

Proceedings to be recorded. Theoretically, off-the-record statements are prohibited. Under R10 of the REE, the testimony of a witness may be presented electronically when allowed by the court after a summary hearing. Voice-activated recording and transcription would fall under this type of evidence.

S3 RIGHTS AND OBLIGATION OF A WITNESS -

SC Guidelines on Pre-trial and Discovery exhort courts to adhere strictly to the “One-Day Examination of Witness” Rule subject to justifiable exceptions. A witness can be compelled to answer to the fact of his prior FINAL conviction for an offense. The answer can be used to impeach the witness or prove his bad moral character but generally not to prove that he committed a similar act as that would run afoul of the similar acts rule.

S6 CROSS-EXAMINATION -

-

-

EXTENT: Matters stated in the direct or connected therewith. PURPOSE: To test the witness’ impartiality, honesty, and accuracy (IHA), and to elicit all important facts bearing upon the issue. Two rules on the scope of cross-examination: American & English. Wordings of S6 R132 show that it is an adoption of the English rule. In 2 instances however the American rule is followed: (CHUA) o S12 R132: Cross-examination of a witness who has been called as an adverseparty witness by the other side or who has been declared as a hostile or unwilling witness upon the initiative of the other side. o S1(d) R115. Cross-examination of the accused in a criminal case. Re-direct examination. Re-examination of the witness in order to explain or supplement his answers given during the cross-examination. On redirect, questions on matters not dealt with during the cross may be allowed by the court in its discretion. Re-cross examination. The witness may be re-crossed on matters stated in the direct and on such other matters as may be allowed at the court’s discretion. Testimony of witness who died before he could be cross-examined must be expunged if there is no showing of delay by party waiting to cross. (Sps. Dela Cruz v. Papa, 8 December 2010, Abad, J.) In this case, the postponement was made by the proponent to give a chance to the witness to recover from his illness but the witness eventually died.

LEADING QUESTION Definition: One which suggests to the witness the answer which the examining party desires. The reason for prohibiting leading questions is that it is virtually the lawyer testifying rather than the witness. When allowed: (PICHUCA) -

On cross. On preliminary (background) matters. Ignorant witness, child of tender years, feeble-minded or deaf-mute witness. (DFIC) Unwilling or hostile witness.

Jurists Bar Review Pointers on Evidence 2015. All rights reserved 2015 by Jurists Review Center, Inc. Unauthorized reproduction, use, or dissemination strictly prohibited and shall be prosecuted to the full extent of the law, including administrative complaints with the Office of the Bar Confidant Supreme Court. Bar Review Pointers on Evidence by Prof. Manuel R. Riguera for Jurists Bar Review Center

26

-

Adverse-party witness, or a director, officer, or managing agent (DOM) of a corporation, partnership, or association which is an adverse party. Child Witness (S20 RECW). Court has discretion to allow leading questions if the same will further the interests of justice.

Misleading question: One which assumes as true a fact not yet testified to by the witness, or contrary to that which he has previously stated. It is never allowed. If asked on direct, the more precise objection is that the question lacks basis. IMPEACHMENT: Means to destroy or put in doubt the credibility of the witness or his testimony. Not necessary to show that witness is lying but that witness may be mistaken in her observation or narration. How adverse party’s witness impeached: (RICC) -

Reputation for honesty, integrity, or truth (HIT) is bad. Inconsistent statement. Contradictory evidence. Conviction. (S11 R132)

Inconsistent Statement (S13 R132) -

-

-

Refers to statements made outside of the case where the witness is testifying. Thus a statement to the police investigator. It does not refer to statements which are made in the same case, in which case they may be used to impeach the witness without laying the predicate. Thus a witness may be impeached by an inconsistent statement made in his answer or deposition taken in the same case without requiring the witness to explain the inconsistency (See S4[a] R23). If the deposition is however taken in another case, the predicate should be laid. Impeachment by inconsistent statement, requires that the predicate be laid. The laying of the predicate is done as follows: (WARE) o If the statements are in writing, they must first be shown to the witness before any question is asked of him. o The statements must be related to him, with the circumstances of the time and place and the persons present. o He must be asked whether he made such statements. o If so, he must be asked to explain the inconsistency. If the predicate is not laid, the impeachment is not complete and the witness has not been impeached effectively. (Pp v. Cortezano, 375 SCRA 95).

Reputation for Honesty, Integrity, and Truth (HIT) is Bad -

-

Note that reputation evidence only may be used and not evidence of particular facts or acts., e.g. of lying. Thus evidence that the witness lied in another court case is not admissible for impeachment purposes. Note however that a witness must answer to the fact of his previous final conviction for an offense. (S3[5] R132).

Contradictory evidence. The purpose is not just to show an inconsistency but to prove outright that the witness is mistaken or lying. Thus if the witness testified that defendant’s car entered the intersection when the stoplight was green in his favor but a video recording showed otherwise, the video recording is contradictory evidence. Predicate need not be laid. Conviction

Jurists Bar Review Pointers on Evidence 2015. All rights reserved 2015 by Jurists Review Center, Inc. Unauthorized reproduction, use, or dissemination strictly prohibited and shall be prosecuted to the full extent of the law, including administrative complaints with the Office of the Bar Confidant Supreme Court. Bar Review Pointers on Evidence by Prof. Manuel R. Riguera for Jurists Bar Review Center

27

While a witness may not be impeached by evidence of particular wrongful acts, it may be shown by the examination of the witness or by the record of the judgment, that he has been convicted of an offense. (S11 R132). The conviction must be final. Note that a witness must answer to the fact of his previous final conviction for an offense. (S3 R132). The conviction may be for any offense, not just for perjury or false testimony. A party may not impeach his own witness, exceptions: (S12 R132) A party may not impeach his own witness. The basis for the rule is that a party vouches for his witness. Exceptions: If the witness is an adverse-party witness or a DOM of a CPA which is an adverse party, or one who has been declared by the court as an unwilling or hostile witness. (UHA). Do not confuse adverse-party witness with adverse party’s witness. A witness may be considered as unwilling or hostile only if so declared by the court upon an adequate showing of his adverse interest, unjustified reluctance to testify, and of his having misled the party into calling him to the witness stand. (ARM). Impeachment of one’s own witness who is UHA Just like impeaching an adverse witness except that impeachment by reputation evidence is not allowed. (ICC). Q Petitioner presented Kenneth Sy as its witness. Stephen Sy was in the courtroom and he listened to Kenneth’s testimony. When Petitioner was about to present Stephen as its next witness, Respondent’s counsel objected to the presentation invoking S15 R132 on exclusion and separation of witnesses. May the trial court disallow the presentation of Stephen as a witness on the ground that he heard the testimony of Kenneth? A No. Without any prior order or at least a motion for exclusion from any of the parties, a court cannot simply allow or disallow the presentation of a witness solely on the ground that the latter heard the testimony of another witness. It is the responsibility of respondent's counsel to protect the interest of his client during the presentation of other witnesses. If respondent actually believed that Kenneth’s testimony would greatly affect that of Stephen's, then respondent's counsel was clearly remiss in his duty to protect the interest of his client when he did not raise the issue of the exclusion of the witness in a timely manner. (Design Sources International v. Eristingcol, 19 February 2014). Live-link TV testimony of a child witness The prosecutor, counsel or guardian ad litem may apply for an order that the testimony of the child be taken in a room outside the courtroom and be televised to the courtroom by live-link TV. Court may motu proprio hear and determine, with notice to the parties, the need for live-link TV testimony. The judge may question the child in camera about his feelings on testifying in the courtroom but not about the issues at trial. Ground: If there is substantial likelihood that the child would suffer trauma from testifying in the presence of the accused, his counsel or the prosecutor as the case may be, which trauma would impair the completeness or truth of the testimony. (S25 RECW) Videotaped deposition of child witness The prosecutor, counsel, or guardian ad litem may apply for an order that a deposition be taken of the child’s testimony and that it be recorded and preserved on videotape. If the judge finds that the child will not be able to testify in open court during the trial, it shall issue the order. The judge shall preside at the videotaped deposition of the child. Objections to deposition testimony or evidence shall be stated and shall be ruled upon at the time of the taking of the deposition. The rights of the accused during trial, especially the right to counsel and to confront and cross-examine the child, shall not be violated during Jurists Bar Review Pointers on Evidence 2015. All rights reserved 2015 by Jurists Review Center, Inc. Unauthorized reproduction, use, or dissemination strictly prohibited and shall be prosecuted to the full extent of the law, including administrative complaints with the Office of the Bar Confidant Supreme Court. Bar Review Pointers on Evidence by Prof. Manuel R. Riguera for Jurists Bar Review Center

28

the deposition. If at the time of the trial the court finds that the child is unable to testify because: (1) there is substantial likelihood that the child would suffer trauma from testifying in the presence of the accused, his counsel or the prosecutor as the case may be, which trauma would impair the completeness or truth of the testimony; or (2) of any reason under S4(c) R23, the court may admit the videotaped deposition in lieu of his testimony. (S27 RECW). AUTHENTICATION AND PROOF OF DOCUMENTS. Authentication in general All kinds of evidence, not just documentary, should be authenticated before it is received in evidence. In the language of trial lawyers, one must “lay a foundation” for the exhibit. Authentication means establishing that the exhibit is in fact what it purports to be. Authentication of documents PUBLIC DOCUMENTS -

-

-

-

-

-

-

Public Documents consist of: (APO) o Written official acts or records of official acts of the sovereign authority, official bodies and tribunals, and public officers (SOP), whether of the Philippines or of a foreign country. o Acknowledged documents (notarial documents), except wills. o Public records of private documents required by law to be entered therein. (S19 R132). Affidavits and sworn certifications are not public documents because they are not acknowledged but merely sworn to before a notary public. Authentication is not required of public documents because they are presumed authentic by virtue of their nature. The presumption may be overthrown. An improperly notarized deed of sale is not a public document which carries the presumption of due execution and authenticity. Hence the evidentiary standard to prove forgery is merely preponderance of evidence, not clear and convincing evidence. The alleged seller’s testimony that his signature was forged taken with the testimony of the buyer that he did not see the seller sign and that he did not acknowledge the document before the notary public was sufficient to prove forgery even without presenting a handwriting expert. (Dela Rama v. Papa, 30 January 2009). Acknowledgment defective where person who appeared before notary public not indicated in the acknowledgment. Hence the document is not considered a public document. (Dycoco v. Orina, 30 July 2010). The fact that Pantranco’s articles of incorporations, memorandum of agreement, and purchase agreement were collected by the PCGG in the course of its investigation of the Marcoses’ alleged ill-gotten wealth do not make the same public records within the purview of S19(c) R132. (Republic v. Marcos-Manotoc, 8 February 2012). The certification as to the income of the deceased, executed by his employer the United States Agency for International Development (USAID) are public documents under S19(a) R132. Hence its authenticity and due execution are already presumed and moreover it is prima facie evidence of the facts stated therein under S23 R132. (Heirs of Ochoa v. G&S Transport, 16 July 2012). The chemistry report showing a positive result of the paraffin test is a public document under S19(a) R132. As a public document, the rule on authentication does not apply. It is admissible in evidence without further proof of its due execution and genuineness; the forensic chemist who made the report need not be presented in court to identify, describe and testify how the report was conducted. (Kummer v Pp, 11 Sep 2013). NSO certification of marriage, death, and that private respondent has record of 2 marriages are public documents pursuant to Article 410 of the Civil Code which provides that the books making up the civil register and all documents relating thereto shall be considered as public documents and shall be prima facie proof of the facts therein. Hence

Jurists Bar Review Pointers on Evidence 2015. All rights reserved 2015 by Jurists Review Center, Inc. Unauthorized reproduction, use, or dissemination strictly prohibited and shall be prosecuted to the full extent of the law, including administrative complaints with the Office of the Bar Confidant Supreme Court. Bar Review Pointers on Evidence by Prof. Manuel R. Riguera for Jurists Bar Review Center

29

there was no need to present the records custodian of the NSO to testify on their authenticity and due execution. (Iwasawa v. Gangan, 11 Sep 2013). Proof of written official acts 1. Official publication thereof, or 2. Copy attested by the officer having the legal custody of the record. If the record is not kept in the Philippines, with a certificate that such officer has the custody. Proof of public record of private documents 1. Original record, or 2. Copy attested by the legal custodian of the record, with an appropriate certificate that such officer has the custody. Public documents as evidence Documents consisting of entries in public records made in the performance of a duty by a public officer are prima facie evidence of the facts stated therein. (S23 R132). This is in effect an exception to the hearsay rule. (See also S44 R130). CENRO and PENRO certifications that land alienable and disposable do not fall within the purview of the first sentence of S23 R132. (Republic v Medida, 13 August 2012). They fall within the purview of the second sentence and are thus proof only of the fact of execution and the date thereof. (Republic v. De Tensuan, 23 October 2013). Proof of lack of record A written statement signed by an officer having the custody of an official record or by his deputy that after diligent search, no record or entry of a specified tenor is found to exist in the records, accompanied by a certificate that the officer has the custody, is admissible as evidence that the records of his office contain no such record or entry. (S28 R132). The certification of the Municipal Civil Registrar of Carmona, Cavite, that marriage license no. 9969967 was issued to another couple and that “it appear that no marriage license was issued to Syed Abbas and Gloria Goo” was sufficient to prove the lack of marriage license. The certification need not stated that a diligent search was made because of the presumption that official duty has been regularly performed. Besides the fact that the MCR stated that license no. 9969967 was issued to another couple shows that she undertook a diligent search. (Abbas v. Abbas, 30 January 2013). Proof of acknowledged documents May be presented in evidence without further proof, the certificate of acknowledgment being prima facie evidence of execution. (S30 R132) Copy not proof unless excepted from the best evidence rule or foundation laid for the presentation of secondary evidence. PRIVATE DOCUMENTS -

Before a private document offered as authentic is received in evidence it should first be authenticated, that is, its due execution and authenticity should be proved by (a) anyone who saw the document executed or written (sew) or (b) evidence of the genuineness of the signature or handwriting of the maker (EGSHaM). (SG). (S20 R132)

-

The genuineness of a person’s handwriting may be proved by: (WC) 1. A witness who has seen the person write. What is written here is a document other than the one sought to be authenticated. If what the witness saw is the execution or writing of the document, the same is authenticated under S20(a) R132.

Jurists Bar Review Pointers on Evidence 2015. All rights reserved 2015 by Jurists Review Center, Inc. Unauthorized reproduction, use, or dissemination strictly prohibited and shall be prosecuted to the full extent of the law, including administrative complaints with the Office of the Bar Confidant Supreme Court. Bar Review Pointers on Evidence by Prof. Manuel R. Riguera for Jurists Bar Review Center

30

2. Or has seen writing purporting to be his upon which the witness has acted or been charged, and has thus acquired knowledge of the handwriting of such person (acak). E.g. a signed instruction for a person to make payment to a particular agent and the statement of account thereafter shows that the payment was credited. 3. A comparison, made by the witness or the court, with writings admitted or treated as genuine by the adverse party or proved (atp) to be genuine to the satisfaction of the judge. (S22 R132). -

If a private document is not being offered as an authentic document but only to prove its existence or the fact of receipt or some other matter, it need not be authenticated. E.g., presentation by the prosecution of a ransom note. o Ancient documents need not be authenticated. An ancient document is a private document more than 30 years old, is produced from a custody in which it would naturally be found if genuine, and is unblemished by any alterations or circumstances of suspicion. CUT (S21 R132) o Under FRE 803(16), statements in an ancient document are excepted from the hearsay rule. In the absence of any similar exception in our Rules of Evidence, such statements should be considered as hearsay if offered to prove the truth of the matter asserted therein. Malayan Insurance Co. v. Phil. Nails Corp., G.R. 138084, 10 April 2002: Respondent sued Petitioner on a marine cargo policy for loss arising from short delivery of steel billets. Petitioner was declared in default and a default judgment rendered in favor of the Respondent. Respondent’s lone witness presented Ms. King who prepared the summary of steel billets received based on the bill of lading and the SGS report. The SC however found that the B/L and the SGS report were not authenticated as Ms King did not see the same executed or written nor was evidence of the genuineness of the handwriting or signature thereon adduced pursuant to S20 R132. Hence the SC said that the summary of steel billets received had no proven real basis.

-

Authentication by circumstantial evidence: Where there is no eyewitness to the execution or writing of a document or no evidence concerning the genuineness of the maker’s signature or handwriting can be obtained, the document may still be authenticated using circumstantial evidence. Examples: o Self-authenticated documents. Where the facts in the writing could only have been known by the writer. For instance the use of code terms likely to be known only by the participants in the underlying events, or by other revealing contents, such as dates, amounts, etc., known only to the participants in the events. (ANTONIO R. BAUTISTA, BASIC EVIDENCE 149 [2004 ed.]). o Reply-authenticated documents. Where the reply of the adverse party refers to and affirms the sending to him and his receipt of the letter in question, a copy of which the proponent is offering in evidence. o Reply-letter rule. Where a reply to a letter indicates that the one writing the reply knew the tenor of the letter, the reply is deemed authenticated.

-

Where party may not object to lack of authentication

-

Estrada v. Desierto, supra. Estrada argued that the Angara Diary was not properly authenticated under S20 R132. The SC said however that “a party who does not deny the genuineness of a proferred instrument may not object that it was not properly identified before it was received in evidence.” Estrada did not deny the authenticity of the Angara Diary despite opportunity to do so. The ruling applies only against a party who had the opportunity and the means to challenge the authenticity of a document. A person who is not a party to the document nor privy thereto is not covered by the rule.

Jurists Bar Review Pointers on Evidence 2015. All rights reserved 2015 by Jurists Review Center, Inc. Unauthorized reproduction, use, or dissemination strictly prohibited and shall be prosecuted to the full extent of the law, including administrative complaints with the Office of the Bar Confidant Supreme Court. Bar Review Pointers on Evidence by Prof. Manuel R. Riguera for Jurists Bar Review Center

31

Where the genuineness and due execution of a document has been expressly or impliedly admitted, as in S8 R8 or under R26, authentication is no longer necessary. AUTHENTICATION OF ELECTRONIC DOCUMENTS -

Before any private electronic document offered as authentic is received in evidence, its authenticity must be proved by any of the following means: o Evidence that it had been digitally signed. o Appropriate security procedures or devices as may be authorized by law or rules have been applied to the document, or o Other evidence showing its reliability and integrity to the satisfaction of the judge. (S2 R5, REE).

Accused argued that an obscene picture sent through a text message was an electronic document that should have been authenticated by a digital signature. SC said that the objection was not raised during the trial and was thus waived. (Ang v. Court of Appeals, 20 April 2010, Abad, J.). Furthermore a picture even if sent through a text message is not an electronic document but a photographic and hence object evidence. SC committed an oversight in this case when it stated that the REE does not apply to criminal actions and proceedings. The SC overlooked SC resolution dated 24 September 2002 which provides that the REE applies to criminal actions and proceedings effective 14 October 2002. OFFER AND OBJECTION When and how to offer evidence: -

-

-

-

Testimonial evidence. At the time the witness is called to testify. The offer is made orally. In cases covered by the Judicial Affidavit Rule, the party presenting the judicial affidavit of his witness in place of direct testimony shall state the purpose of such testimony at the start of the presentation of the witness. Documentary evidence. After the presentation of a party’s testimonial evidence and before he rests his case. The offer shall be done orally unless allowed by the court to be done in writing. In cases covered by the JAR, upon the termination of the testimony of his last witness, a party shall immediately make an oral offer of evidence of his documentary or object exhibits, piece by piece, in their chronological order, stating the purpose/s for which he offers the particular exhibit. Object evidence. Same as documentary evidence, unless it cannot be submitted to the court in which case at the time it is presented to the court’s senses. In cases covered by the JAR, the same rule for offering documentary evidence applies. The proponent should state the purpose/s for which he is offering the evidence.

Reason: So that the court may consider the evidence for the purpose for which it was offered. Evidence not offered cannot be considered by the Court. Mere identification of documents and the marking thereof as exhibits does not confer any evidentiary weight on the documents unless these are formally offered. (Heirs of Cruz-Zamora v. Multiwood Inter. Inc., 19 Jan 09). OBJECTIONS When to object: -

Testimonial evidence: At two points. One when the offer is made, and second, when an objectionable question is asked of the witness. Objection must be made immediately after the offer is made. Objection to a question propounded in the course of the oral examination of a witness shall be made as soon as the grounds therefore shall become reasonably apparent.

Jurists Bar Review Pointers on Evidence 2015. All rights reserved 2015 by Jurists Review Center, Inc. Unauthorized reproduction, use, or dissemination strictly prohibited and shall be prosecuted to the full extent of the law, including administrative complaints with the Office of the Bar Confidant Supreme Court. Bar Review Pointers on Evidence by Prof. Manuel R. Riguera for Jurists Bar Review Center

32

-

-

Documentary and object evidence. When the document is offered in evidence. So objecting when the document is merely being marked and identified is premature. When the evidence is offered orally, objection must be made immediately after the offer. When the evidence is offered in writing, objection must be made within 3 days from notice, unless a different period is allowed by the court. UNDER THE JAR: The party presenting the judicial affidavit of his witness in place of direct testimony shall state the purpose of such testimony at the start of the presentation of the witness. The adverse party may then move to disqualify the witness or to strike out his affidavit or any of the answers on the ground of inadmissibility.

General objection cf. specific objection. A specific objection is one which specifies its ground while a general objection does not specify any ground or uses the broadside or shotgun phrase, “irrelevant and incompetent.” The rules prohibit a general objection. S36 R132 states that “in any case, the grounds for the objection must be specified.” MOTION TO STRIKE OUT (S39 R132) Remedy of a party where the objectionable feature was not apparent from the question or where the witness answered instantly without giving the lawyer a chance to object. Also under the JAR, the motion to strike out or disqualify takes the place of the objection. CONTINUING OBJECTION When a party has previously objected to a question, whether sustained or overruled, and succeeding questions are of the same class as those to which objection has been made, it shall not be necessary to repeat the objection, it being sufficient for the adverse party to record his continuing objection to such class of questions. (S37 R132). RULING -

-

-

The court should rule immediately on the objection although it can where necessary take a reasonable time to study the matter but the ruling should always be made during the trial. The court need not state the ground for its ruling except when the objection is based on 2 or more grounds in which case a ruling sustaining the objection must state on what ground/s it is based. No need to except to ruling in order to challenge it on appeal. Note however that if the ruling excludes offered evidence, the proponent must make an offer of proof.

TENDER OF EXCLUDED EVIDENCE OR OFFER OF PROOF (S40 R132) -

-

If documents or things offered in evidence are excluded by the court, the offeror may have the same attached to or made part of the record. If the evidence excluded is oral, the offeror may state for the record the name and other personal circumstances of the witness and the substance of the proposed testimony. (NPS) The purpose of making an offer of proof is to allow the appellate court to assess whether the trial court’s exclusion of the evidence is proper.

Q Viveca brought against her husband Philip Yu (Petitioner) an action for legal separation and dissolution of conjugal partnership on the grounds of marital infidelity and physical abuse. During trial Viveca moved for the issuance of a subpoena duces tecum and ad testificandum to Insular Life officers to compel production of the insurance application and policy of a person suspected to be Philip’s illegitimate child. The trial court denied the motion on the ground that under Circular Letter No. 11-2000 issued by the Insurance Commission, insurance companies/agents are prohibited from divulging confidential and privileged information pertaining to insurance policies.

Jurists Bar Review Pointers on Evidence 2015. All rights reserved 2015 by Jurists Review Center, Inc. Unauthorized reproduction, use, or dissemination strictly prohibited and shall be prosecuted to the full extent of the law, including administrative complaints with the Office of the Bar Confidant Supreme Court. Bar Review Pointers on Evidence by Prof. Manuel R. Riguera for Jurists Bar Review Center

33

Viveca filed a motion for reconsideration of the denial but this was denied. She filed a petition for certiorari under R65 with the Court of Appeals. In the trial court, she filed her formal offer of rebuttal exhibits with tender of excluded evidence, referring to the insurance policy and application. The Petitioner argues that Viveca’s tender of excluded evidence mooted her R65 petition. a) Was the R65 petition of Viveca rendered moot and academic when she made a tender of excluded evidence before the trial court? b) Are the contents of an insurance policy and application confidential and privileged? A a) No the R65 petition of Viveca was not rendered moot and academic when she made a tender of excluded evidence before the trial court. Under S40 R132 before tender of excluded evidence may be made the evidence must be formally offered to the court and denied admission by it. Here the insurance policy and application were not formally offered to the trial court. Hence the tender of proof was not an adequate remedy which precludes the filing of a petition for certiorari. b) No the contents of an insurance policy and application are not privileged and confidential. The Insurance Commission itself has issued an opinion that Circular Letter No. 11-2000 was not designed to obstruct lawful court orders. (Yu v. CA, G.R. 154115, 29 Nov 05).

-oOo-

Jurists Bar Review Pointers on Evidence 2015. All rights reserved 2015 by Jurists Review Center, Inc. Unauthorized reproduction, use, or dissemination strictly prohibited and shall be prosecuted to the full extent of the law, including administrative complaints with the Office of the Bar Confidant Supreme Court. Bar Review Pointers on Evidence by Prof. Manuel R. Riguera for Jurists Bar Review Center

34

View more...

Comments

Copyright ©2017 KUPDF Inc.
SUPPORT KUPDF